Sie sind auf Seite 1von 110

INSURANCE LAW

CONSTRUCTION OF THE INSURANCE CODE:

1. Insular v Ebrado G.R. No. L-44059 October 28, 1977

Facts:

Cristor Ebrado was issued by The Life Assurance Co., Ltd., a policy for
P5,882.00 with a rider for Accidental Death. He designated Carponia T.
Ebrado as the revocable beneficiary in his policy. He referred to her as
his wife.

Cristor was killed when he was hit by a failing branch of a tree. Insular
Life was made liable to pay the coverage in the total amount of
P11,745.73, representing the face value of the policy in the amount of
P5,882.00 plus the additional benefits for accidental death.

Carponia T. Ebrado filed with the insurer a claim for the proceeds as
the designated beneficiary therein, although she admited that she and
the insured were merely living as husband and wife without the benefit
of marriage.

Pascuala Vda. de Ebrado also filed her claim as the widow of


the deceased insured. She asserts that she is the one entitled to the
insurance proceeds.

Insular commenced an action for Interpleader before the trial court as


to who should be given the proceeds. The court declared Carponia
as disqualified.

Issue: WON a common-law wife named as beneficiary in the life


insurance policy of a legally married man can claim the proceeds in
case of death of the latter?

Held: No. Petition

Ratio:

Section 50 of the Insurance Act which provides that "the insurance


shall be applied exclusively to the proper interest of the person in
whose name it is made"
The word "interest" highly suggests that the provision refers only to
the "insured" and not to the beneficiary, since a contract of insurance
is personal in character. Otherwise, the prohibitory laws against illicit
relationships especially on property and descent will be rendered
nugatory, as the same could easily be circumvented by modes of
insurance.

When not otherwise specifically provided for by the Insurance Law, the
contract of life insurance is governed by the general rules of the civil
law regulating contracts. And under Article 2012 of the same Code,
any person who is forbidden from receiving any donation under Article
739 cannot be named beneficiary of a fife insurance policy by the
person who cannot make a donation to him. Common-law spouses are
barred from receiving donations from each other.

Article 739 provides that void donations are those made between
persons who were guilty of adultery or concubinage at the time
of donation.

There is every reason to hold that the bar in donations between


legitimate spouses and those between illegitimate ones should be
enforced in life insurance policies since the same are based on similar
consideration. So long as marriage remains the threshold of family
laws, reason and morality dictate that the impediments imposed upon
married couple should likewise be imposed upon extra-marital
relationship.

A conviction for adultery or concubinage isn’t required exacted before


the disabilities mentioned in Article 739 may effectuate. The article
says that in the case referred to in No. 1, the action for declaration of
nullity may be brought by the spouse of the donor or donee; and the
guilty of the donee may be proved by preponderance of evidence in
the same action.

The underscored clause neatly conveys that no criminal conviction for


the offense is a condition precedent. The law plainly states that the
guilt of the party may be proved “in the same acting for declaration of
nullity of donation.” And, it would be sufficient if evidence
preponderates.
The insured was married to Pascuala Ebrado with whom she has six
legitimate children. He was also living in with his common-law wife
with whom he has two children.

2.Qua Chee Gan v. Law Union Rock - Breach of Warranty

Facts:

> Qua Chee Gan, a merchant, owned 4 warehouses in Albay which


were used for the storage or copra and hemp in which the appelle
deals with exclusively.

> The warehouses together with the contents were insured with Law
Union since 1937 and the loss made payable to PNB as mortgagee of
the hemp and copra.

> A fire of undetermined cause broke out in July 21, 1940 and lasted
for almost 1 whole week.

> Bodegas 1, 3, and 4 including the merchandise stored were


destroyed completely.

> Insured then informed insurer of the unfortunate event and


submitted the corresponding fire claims, which were later reduced to
P370T.

> Insurer refused to pay claiming violations of the warranties and


conditions, filing of fraudulent claims and that the fire had been
deliberately caused by the insured.

> Insured filed an action before CFI which rendered a decision in


favor of the insured.

Issues and Resolutions:

(1) Whether or not the policies should be avoided for the reason that
there was a breach of warranty.

Under the Memorandum of Warranty, there should be no less than 1


hydrant for each 150 feet of external wall measurements of the
compound, and since bodegas insured had an external wall per meter
of 1640 feet, the insured should have 11 hydrants in the compound.
But he only had

2.Even so, the insurer is barred by estoppel to claim violation of the


fire hydrants warranty, because knowing that the number of hydrants
it demanded never existed from the very beginning, appellant
nevertheless issued the policies subject to such warranty and received
the corresponding premiums. The insurance company was aware,
even before the policies were issued, that in the premises there were
only 2 hydrants and 2 others were owned by the Municipality, contrary
to the requirements of the warranties in question.

It should be close to conniving at fraud upon the insured to allow the


insurer to claim now as void the policies it issued to the insured,
without warning him of the fatal defect, of which the insurer was
informed, and after it had misled the insured into believing that the
policies were effective.

Accdg to American Jurisprudence: It is a well-settled rule that the


insurer at the time of the issuance of a policy has the knowledge of
existing facts, which if insisted on, would invalidate the contract from
its very inception, such knowledge constitutes a waiver of conditions in
the contract inconsistent with known facts, and the insurer is stopped
thereafter from asserting the breach of such conditions. The reason
for the rule is: To allow a company to accept one’s money for a policy
of insurance which it knows to be void and of no effect, though it
knows as it must that the insured believes it to be valid and binding is
so contrary to the dictates of honesty and fair dealing, as so closely
related to positive fraud, as to be abhorrent to fair-minded men. It
would be to allow the company to treat the policy as valid long enough
to get the premium on it, and leave it at liberty to repudiate it the next
moment.

Moreover, taking into account the well-known rule that ambiguities or


obscurities must strictly be interpreted against the party that cause
them, the memorandum of warranty invoked by the insurer bars the
latter from questioning the existence of the appliances called for, since
its initial expression “the undernoted appliances for the extinction of
fire being kept on the premises insured hereby..” admits of the
interpretation as an admission of the existence of such appliances
which insurer cannot now contradict, should the parole evidence apply.

(2) Whether or not the insured violated the hemp warranty provision
against the storage of gasoline since insured admitted there were 36
cans of gasoline in Bodega 2 which was a separate structure and not
affected by the fire.

It is well to note that gasoline is not specifically mentioned among the


prohibited articles listed in the so-called hemp warranty. The clause
relied upon by the insurer speaks of “oils”. Ordinarily, oils mean
lubricants and not gasoline or kerosene. Here again, by reason of the
exclusive control of the insurance company over the terms of the
contract, the ambiguity must be held strictly against the insurer and
liberally in favor of the insured, specially to avoid a forfeiture.

Furthermore, the gasoline kept was only incidental to the insured’s


business. It is a well settled rule that keeping of inflammable oils in
the premises though prohibited by the policy does NOT void it if such
keeping is incidental to the business. Also, the hemp warranty forbade
the storage only in the building to which the insurance applies, and/or
in any building communicating therewith; and it is undisputed that no
gasoline was stored in the burnt bodegas and that Bodega No. 2 which
was where the gasoline was found stood isolated from the other
bodegas.

Qua Chee Gan V. Law Union And Rock Insurance Co., Ltd. (1955)

Lessons Applicable: Ambiguous Provisions Interpreted Against Insurer


(Insurance)

FACTS:

 Qua Chee Gan, a merchant of Albay, owned four bodegas which


he insured with Law Union & Rock Insurance Co., Ltd (Law
Union) since 1937 and the lose made payable to the Philippine
National Bank (PNB) as mortgage of the hemp and crops, to the
extent of its interest

 July 21, 1940 morning: fire broke out in bodegas 1,2 and 4
which lasted for almost a week.
 Qua Chee Gan informed Law Union by telegram

 Law Union rejected alleging that it was a fraudulent claim that


the fire had been deliberately caused by the insured or by other
persons in connivance with him

 Que Chee Gan, with his brother, Qua Chee Pao, and some
employees of his, were indicted and tried in 1940 for the crime
of arson but was subsequently acquitted

 During the pendency of the suit, Que Chee Gan paid PNB

 Law Union states that ff. assignment of errors:

 1. memo of warranty requires 11 hydrants instead of 2

 2. violation of hemp warranty against storage of gasoline


since it prohibits oils

 3. fire was due to fraud

 4. burned bodegas could not possibly have contained the


quantities of copra and hemp stated in the fire claims

ISSUE: W/N Qua Chee Gan should be allowed to claim.

HELD: YES. Affirmed.

 1. It is a well settled rule of law that an insurer which with


knowledge of facts entitling it to treat a policy as no longer in
force, receives and accepts a preium on the policy, estopped to
take advantage of the forfeiture

 2. oils (animal and/or vegetable and/or mineral and/or their


liquid products having a flash point below 300o Fahrenheit", and
is decidedly ambiguous and uncertain; for in ordinary parlance,
"Oils" mean "lubricants" and not gasoline or kerosene

 by reason of the exclusive control of the insurance


company over the terms and phraseology of the contract,
the ambiguity must be held strictly against the insurer
and liberraly in favor of the insured, specially to avoid a
forfeiture
 3. trial Court found that the discrepancies were a result of the
insured's erroneous interpretation of the provisions of the
insurance policies and claim forms, caused by his imperfect
knowledge of English, and that the misstatements were
innocently made and without intent to defraud.

 4. Similarly, the 20 per cent overclaim on 70 per cent of the


hemo stock, was explained by the insured as caused by his
belief that he was entitled to include in the claim his expected
profit on the 70 per cent of the hemp, because the same was
already contracted for and sold to other parties before the fire
occurred

3.Ty v. Filipinas Compañia de Seguros - Insurance Policy

Facts:

> Ty was employed as a mechanic operator by Braodway Cotton


Factory at Grace Park, Caloocan.

> In 1953, he took personal accident policies from 7 insurance


companies (6 defendants), on different dates, effective for 12 mos.

> On Dec. 24. 1953, a fire broke out in the factory were Ty was
working. A hevy object fell on his hand when he was trying to put out
the fire.

> From Dec. 1953 to Feb. 6, 1954 Ty received treatment at the Nat’l
Orthopedic Hospital for six listed injuries. The attending surgeon
certified that these injuries would cause the temporary total disability
of Ty’s left hand.

> Insurance companies refused to pay Ty’s claim for compensation


under the policies by reason of said disability of his left hand. Ty filed
a complaint in the municipal court who decided in his favor.

> CFI reversed on the ground that under the uniform terms of the
policies, partial disability due to loss of either hand of the insured, to
be compensable must be the result of amputation.

Issue:
Whether or not Ty should be indemnified under his accident policies.

Held.

NO.

SC already ruled in the case of Ty v. FNSI that were the insurance


policies define partial disability as loss of either hand by amputation
through the bones of the wrist, the insured cannot recover under said
policies for temporary disability of his left hand caused by the fractures
of some fingers. The provision is clear enough to inform the party
entering into that contract that the loss to be considered a disability
entitled to indemnity, must be severance or amputation of the affected
member of the body of the insured.

Ty V. First National Surety And Assurance Co., Inc. (1961)

Lessons Applicable: Clear Provision Given Ordinary Meaning


(Insurance)
FACTS:
 2 months prior to December 24, 1953: Diosdado C. Ty, employed
as operator mechanic foreman in the Broadway Cotton
Factory insured himself in 18 local insurance companies
with Broadway Cotton Factory as his beneficiary
 December 24, 1953: fire broke out at the Broadway Cotton Factory
where Ty, fighting his way out, injured his left hand by a heavy
object.
 He was brought to the Manila Central University hospital, and after
receiving first aid there, he went to the National Orthopedic
Hospital for treatment of his injuries.
 His injuries caused temporary total disability on his left hand so he
filed a claim against all defendants who rejected the claim
reasoning that there it was not covered in his policy because there
was no severance of amputation of the left hand
 Trial Court: absolved the defendants
ISSUE: W/N Ty can claim
HELD: NO. Affirmed.
 can not go beyond the clear and express conditions of the
insurance policies, all of which define partial disability as loss of
either hand by amputation through the bones of the wrist
 Note that the disability of plaintiff's hand was merely temporary,
having been caused by fracture of the index, the middle and the
fourth fingers of the left hand
 agreement contained in the insurance policies is the law between
the parties

4.Del Rosario v. Equitable Insurance - Life Insurance Policy

Facts:

> Equitable Insurance issued a life Insurance policy to del Rosario


binding itself to pay P1,000 to P3,000 as indemnity.

> Del Rosario died in a boating accident. The heirs filed a claim and
Equitable paid them P1,000.

> The heir filed a complaint for recovery of the balance of P2,000,
claiming that the insurere should pay him P3,000 as stated in the
policy.

Issue:

Whether or not the heir is entitled to recover P3,000.

Held: YES.

Generally accepted principles or ruling on insurance, enunciate that


where there is an ambiguity with respect to the terms and conditions
of the policy, the same shall be resolved against the one responsible
thereof. The insured has little, if any, participation in the preparation
of the policy. The interpretation of obscure stipulations in a contract
should not favor the party who cause the obscurity

Del Rosario V. Equitable Ins. And Casualty Co., Inc. (1963)


Lessons Applicable: Ambiguous Provisions Interpreted Against Insurer
(Insurance)
FACTS:
 April 13, 1957: Simeon del Rosario, father of the insured who died
from drowning filed a claim for payment with Equitable Ins. and
Casualty Co., Inc. but it refused to pay more than P1,000 php so a
case was filed with the RTC for the P2,000 balance stating that
under the policy they are entitled to P1,000 to P3,000 as indemnity
 RTC: entitled to recover P3,000 - policy does not positively state
any definite amount, there is an ambiguity in this respect in the
policy, which ambiguity must be interpreted in favor of the insured
and strictly against the insurer so as to allow greater indemnity
ISSUE: W/N Simeon is entitled to recover P3,000
HELD: YES.
 terms in an insurance policy, which are ambiguous, equivocal or
uncertain are to be construed strictly against, the insurer, and
liberally in favor of the insured so as to effect the dominant
purpose of indemnity or payment to the insured, especially where a
forfeiture is involved
 reason for this rule is that the "insured usually has no voice in the
selection or arrangement of the words employed and that the
language of the contract is selected with great care and
deliberation by expert and legal advisers employed by, and acting
exclusively in the interest of, the insurance company

5.Misamis Lumber v. Capital Insurance - Insurance Policy

Facts:

> Misamis lumber insured it’s motor car for P14T with Capital
Insurance. The policy stipulated that the insured may authorize the
repair of the vehicle necessitated by damage and the liability of the
insured is limited to 150.
> Car met an accident and was repaired by Morosi Motors at a total
cost of P302.27. Misamis made a report of the accident to Capital who
refused to pay the cost of the repairs.

Issue:

Whether or not the insurer is liable for the total amount of the repair.

Held: NO.

The insurance policy stipulated that if it is the insured who authorized


the repair, the liability of the insurer is limited to 150. The literal
meaning of the stipulation must control, it being the actual contract,
expressly and plainly provided for in the policy.

Misamis Lumber Corp. V. Capital Ins. And Surety Co., Inc. (1966)

Lessons Applicable: Judicial Construction Cannot Alter Terms


(Insurance)
FACTS:
 Misamis Lumber Corporation (Misamis), formerly Lanao Timber
Mills, Inc., insured its Ford Falcon motor car with Capital Insurance
& Surety Company (Capital)
 November 25, 1961 11 pm: The car broke when it hit a hollow
block lying alongside the water hole which the driver did not see
because the on-coming car did not dim its light
 The car was towed and repaired by Morosi Motors costing P302.27
 November 29, 1961: After the repairs were made, Misamis made a
report to Capital who only admits liability of P150
 CFI: paragraph 4 of the policy is clear and specific and leaves no
room for interpretation that the repair liability is limited to P150
ISSUE: W/N Misamis is entitled to an amount exceeding P150
HELD: NO.
 insurance contract may be rather onerous (one-sided) but that in
itself does not justify the abrogation of its express terms, terms
which the insured accepted or adhered to and which is the law
between the contracting parties
6.Verendia v. CA - Insurance Policy

Facts:

> Fidelity and Surety Insurance Company (Fidelity) issued Fire


Insurance Policy No. F-18876 effective between June 23, 1980 and
June 23, 1981 covering Rafael (Rex) Verendia's residential in the
amount of P385,000.00. Designated as beneficiary was the Monte de
Piedad & Savings Bank.

> Verendia also insured the same building with two other companies,
namely, The Country Bankers Insurance for P56,000.00 and The
Development Insurance for P400,000.00.

> While the three fire insurance policies were in force, the insured
property was completely destroyed by fire.

> Fidelity appraised the damage amounting to 385,000 when it was


accordingly informed of the loss. Despite demands, Fidelity refused
payment under its policy, thus prompting Verendia to file a complaint
for the recovery of 385,000

> Fidelity, averred that the policy was avoided by reason of over-
insurance, that Verendia maliciously represented that the building at
the time of the fire was leased under a contract executed on June 25,
1980 to a certain Roberto Garcia, when actually it was a Marcelo
Garcia who was the lessee.

Issue:

Whether or not Verendia can claim on the insurance despite the


misrepresentation as to the lessee and the overinsurance.

Held:NO

The contract of lease upon which Verendia relies to support his claim
for insurance benefits, was entered into between him and one Robert
Garcia, a couple of days after the effectivity of the insurance policy.
When the rented residential building was razed to the ground, it
appears that Robert Garcia was still within the premises. However,
according to the investigation by the police, the building appeared to
have "no occupants" and that Mr. Roberto Garcia was "renting on the
otherside of said compound" These pieces of evidence belie Verendia's
uncorroborated testimony that Marcelo Garcia whom he considered as
the real lessee, was occupying the building when it was burned.

Ironically, during the trial, Verendia admitted that it was not Robert
Garcia who signed the lease contract but it was Marcelo Garcia cousin
of Robert, who had also been paying the rentals all the while.
Verendia, however, failed to explain why Marcelo had to sign his
cousin's name when he in fact he was paying for the rent and why he
(Verendia) himself, the lessor, allowed such a ruse. Fidelity's
conclusions on these proven facts appear, therefore, to have sufficient
bases: Verendia concocted the lease contract to deflect responsibility
for the fire towards an alleged "lessee", inflated the value of the
property by the alleged monthly rental of P6,500) when in fact, the
Provincial Assessor of Rizal had assessed the property's fair market
value to be only P40,300.00, insured the same property with two other
insurance companies for a total coverage of around P900,000, and
created a dead-end for the adjuster by the disappearance of Robert
Garcia.

Basically a contract of indemnity, an insurance contract is the law


between the parties. Its terms and conditions constitute the measure
of the insurer's liability and compliance therewith is a condition
precedent to the insured's right to recovery from the. As it is also a
contract of adhesion, an insurance contract should be liberally
construed in favor of the insured and strictly against the insurer
company which usually prepares it

Considering, however, the foregoing discussion pointing to the fact


that Verendia used a false lease contract to support his claim under
Fire Insurance Policy, the terms of the policy should be strictly
construed against the insured. Verendia failed to live by the terms of
the policy, specifically Section 13 thereof which is expressed in terms
that are clear and unambiguous, that all benefits under the policy shall
be forfeited "if the claim be in any respect fraudulent, or if any false
declaration be made or used in support thereof, or if any fraudulent
means or devises are used by the Insured or anyone acting in his
behalf to obtain any benefit under the policy". Verendia, having
presented a false declaration to support his claim for benefits in the
form of a fraudulent lease contract, he forfeited all benefits therein by
virtue of Section 13 of the policy in the absence of proof that Fidelity
waived such provision

There is also no reason to conclude that by submitting the subrogation


receipt as evidence in court, Fidelity bound itself to a "mutual
agreement" to settle Verendia's claims in consideration of the amount
of P142,685.77. While the said receipt appears to have been a filled-
up form of Fidelity, no representative of Fidelity had signed it. It is
even incomplete as the blank spaces for a witness and his address are
not filled up. More significantly, the same receipt states that Verendia
had received the aforesaid amount. However, that Verendia had not
received the amount stated therein, is proven by the fact that
Verendia himself filed the complaint for the full amount of P385,000.00
stated in the policy. It might be that there had been efforts to settle
Verendia's claims, but surely, the subrogation receipt by itself does not
prove that a settlement had been arrived at and enforced. Thus, to
interpret Fidelity's presentation of the subrogation receipt in evidence
as indicative of its accession to its "terms" is not only wanting in
rational basis but would be substituting the will of the Court for that of
the partie

Verendia V CA G.R. No. 75605 January 22, 1993

G.R. No. 75605 January 22, 1993


Lessons Applicable: Exception to Ambiguous Provisions Interpreted
Against Insurer (Insurance)
FACTS:
 Rafael (Rex) Verendia's residential building was insured
with Fidelity and Surety Insurance Company, Country Bankers
Insurance and Development Insurance with Monte de Piedad &
Savings Bank as beneficiary
 December 28, 1980 early morning: the building was completely
destroyed by fire
 Fidelity refused the claim stating that there was
a misrepresentation since the lessee was not Roberto Garcia
but Marcelo Garcia
 trial court: favored Fidelity
 CA: reversed
ISSUE: W/N there was false declaration which would forfeit his
benefits under Section 13 of the policy
HELD: YES.
 Section 13 thereof which is expressed in terms that are clear and
unambiguous, that all benefits under the policy shall be forfeited "If
the claim be in any respect fraudulent, or if any false declaration be
made or used in support thereof, or if any fraudulent means or
devises are used by the Insured or anyone acting in his behalf to
obtain any benefit under the policy"
 Robert Garcia then executed an affidavit before the National
Intelligence and Security Authority (NISA) to the effect that he was
not the lessee of Verendia's house and that his signature on the
contract of lease was a complete forgery.
 Worse yet, by presenting a false lease contract, Verendia,
reprehensibly disregarded the principle that insurance contracts
are uberrimae fidae and demand the most abundant good faith

THE CONTRACT OF INSURANCE:ITS ELEMENTS

Philamlife v. Ansaldo - Jurisdiction of the Insurance Commissioner

Facts:

> Ramon M. Paterno sent a letter-complaint to the Insurance


Commissioner alleging certain problems encountered by agents,
supervisors, managers and public consumers of the Philamlife as a
result of certain practices by said company.
> Commissioner requested petitioner Rodrigo de los Reyes, in his
capacity as Philamlife's president, to comment on respondent Paterno's
letter.

> The complaint prays that provisions on charges and fees stated in
the Contract of Agency executed between Philamlife and its agents, as
well as the implementing provisions as published in the agents'
handbook, agency bulletins and circulars, be declared as null and void.
He also asked that the amounts of such charges and fees already
deducted and collected by Philamlife in connection therewith be
reimbursed to the agents, with interest at the prevailing rate reckoned
from the date when they were deducted

> Manuel Ortega, Philamlife's Senior Assistant Vice-President and


Executive Assistant to the President, asked that the Commissioner first
rule on the questions of the jurisdiction of the Insurance Commissioner
over the subject matter of the letters-complaint and the legal standing
of Paterno.

> Insurance Commissioner set the case for hearing and sent
subpoena to the officers of Philamlife. Ortega filed a motion to quash
the subpoena alleging that the Insurance company has no jurisdiction
over the subject matter of the case and that there is no complaint
sufficient in form and contents has been filed.

> The motion to quash was denied.

Issue:

Whether or not the insurance commissioner had jurisdiction over the


legality of the Contract of Agency between Philamlife and its agents.

Held:

No, it does not have jurisdiction.

The general regulatory authority of the Insurance Commissioner is


described in Section 414 of the Insurance Code, to wit:

"The Insurance Commissioner shall have the duty to see that all laws
relating to insurance, insurance companies and other insurance
matters, mutual benefit associations and trusts for charitable uses are
faithfully executed and to perform the duties imposed upon him by this
Code, . . . ."

On the other hand, Section 415 provides:

"In addition to the administrative sanctions provided elsewhere in this


Code, the Insurance Commissioner is hereby authorized, at his
discretion, to impose upon insurance companies, their directors and/or
officers and/or agents, for any willful failure or refusal to comply with,
or violation of any provision of this Code, or any order, instruction,
regulation or ruling of the Insurance Commissioner, or any commission
of irregularities, and/or conducting business in an unsafe or unsound
manner as may be determined by the Insurance Commissioner, the
following:

a) fines not in excess of five hundred pesos a day; and

b) suspension, or after due hearing, removal of directors and/or


officers and/or agents."

A plain reading of the above-quoted provisions show that the


Insurance Commissioner has the authority to regulate the business of
insurance, which is defined as follows:

"(2) The term 'doing an insurance business' or 'transacting an


insurance business,' within the meaning of this Code, shall include (a)
making or proposing to make, as insurer, any insurance contract; (b)
making, or proposing to make, as surety, any contract of suretyship as
a vocation and not as merely incidental of the surety; (c) doing any
kind of business, including a reinsurance business, specifically
recognized as constituting the doing of an insurance business within
the meaning of this Code; (d) doing or proposing to do any business in
substance equivalent to any of the foregoing in a manner designed to
evade the provisions of this Code. (Insurance Code, Sec. 2 [2])

Since the contract of agency entered into between Philamlife and its
agents is not included within the meaning of an insurance business,
Section 2 of the Insurance Code cannot be invoked to give jurisdiction
over the same to the Insurance Commissioner. Expressio unius est
exclusio alterius.

Phil. American Life Insurance Company V. Ansaldo (1994)

Lessons Applicable: Doing an Insurance Business (Insurance)

FACTS:
 Ramon M. Paterno, Jr. sent a letter dated April 17, 1986
to Insurance Commissioner alleging certain problems encountered
by agents, supervisors, managers and public consumers of the
Philippine American Life Insurance Company (Philamlife)
 During the hearing Ramon stated that the contract of agency is
illegal
 Philamlife through its president De los Reyes contended that the
Insurance Commissioner as a quasi-judicial body cannot rule on the
matter
ISSUE:
1. W/N the Insurance Commissioner has the authority to regulate the
business of insurance - YES
2. W/N the business of insurance covers the contract of agency - NO

HELD: petition is GRANTED


1. YES.

Insurance Code
Sec. 414
Sec. 414. The Insurance Commissioner shall have the duty to see
that all laws relating to insurance, insurance companies and other
insurance matters, mutual benefit associations, and trusts for
charitable uses are faithfully executed and to perform the duties
imposed upon him by this Code, and shall, notwithstanding any
existing laws to the contrary, have sole and exclusive authority to
regulate the issuance and sale of variable contracts as defined in
section two hundred thirty-two and to provide for the licensing of
persons selling such contracts, and to issue such reasonable rules
and regulations governing the same.

The Commissioner may issue such rulings, instructions, circulars,


orders and decision as he may deem necessary to secure the
enforcement of the provisions of this Code, subject to the approval
of the Secretary of Finance. Except as otherwise specified, decisions
made by the Commissioner shall be appealable to the Secretary of
Finance.
Sec. 415
Sec. 415. In addition to the administrative sanctions provided
elsewhere in this Code, the Insurance Commissioner is hereby
authorized, at his discretion, to impose upon the insurance
companies, their directors and/or officers and/or agents, for any
willful failure or refusal to comply with, or violation of any provision
of this Code, or any order, instruction, regulation, or ruling of the
Insurance Commissioner, or any commission or irregularities, and/or
conducting business in an unsafe or unsound manner as may be
determined by the Insurance Commissioner, the following:

(a) fines not in excess of five hundred pesos a day; and


(b) suspension, or after due hearing, removal of directors and/or
officers and/or agents.

 Insurance Commissioner has the authority to regulate the business


of insurance
2. NO.
 power does not cover the relationship affecting the insurance
company and its agents but is limited to adjudicating claims and
complaints filed by the insured against the insurance company
 While the subject of Insurance Agents and Brokers is discussed
under Chapter IV, Title I of the Insurance Code, the provisions of
said Chapter speak only of the licensing requirements and
limitations imposed on insurance agents and brokers.
 Great Pacific Life Assurance Corporation v. Judico, 180 SCRA 445
(1989):
 insurance company may have two classes of agents who sell its
insurance policies:
 (1) salaried employees who keep definite hours and work under the
control and supervision of the company - governed by the Contract
of Employment and the provisions of the Labor Code
 (2) registered representatives, who work on commission basis.
- governed by the Contract of Agency and the provisions of the
Civil Code on the Agency

Filipinas Cia de Seguros v. Christern Huenfeld & Co. - Enemy


Corporation

Facts:

> Oct. 1, 1941, Domestic Corp Christern, after payment of the


premium, obtained from Filipinas, fire policy no. 29333 for P100T
covering merchandise contained in a building located in Binondo.

> On Feb. 27, 1942, during the Jap occupation, the building and the
insured merchandise were burned. Christern submitted to Filipinas its
claim.

> Salvaged goods were sold and the total loss of Christern was P92T.

> Filipinas denied liability on the ground that Christern was an enemy
corporation and cannot be insured.

Issue:

Whether or not Filipinas is liable to Christern, Huenfeld & Co.

Held:NO.

Majority of the stockholders of Christern were German subjects. This


being so, SC ruled that said corporation became an enemy corporation
upon the war between the US and Germany. The Phil Insurance Law
in Sec. 8 provides that anyone except a public enemy may be
insured. It stands to reason that an insurance policy ceases to be
allowable as soon as an insured becomes a public enemy.
The purpose of the war is to cripple the power ad exhaust the
resources of the enemy, and it is inconsistent that one country should
destroy its enemy property and repay in insurance the value of what
has been so destroyed, or that it should in such manner increase the
resources of the enemy or render it aid.

All individuals who compose the belligerent powers, exist as to each


other, in a state of utter exclusion and are public enemies. Christern
having become an enemy corporation on Dec. 10. 1941, the insurance
policy issued in his favor on Oct. 1, 1941 by Filipinas had ceased to be
valid and enforceable, and since the insured goods were burned after
Dec. 10, 1941, and during the war, Christern was NOT entitled to any
indemnity under said policy from Filipinas.

Elementary rules of justice require that the premium paid by Christern


for the period covered by the policy from Dec. 10, 1941 should be
returned by Filipinas.

Filipinas Compania De Seguros V. Christern Henefeld And Co. (1951)

Lessons Applicable: Disqualification: Public Enemy (Insurance)


FACTS:
 October 1, 1941: Christern Huenefeld and co., inc. (Christern), a
company whose major stockholders are German, paid P1M and
obtained a fire policy fromFilipinas Cia. de Seguros (Filipinas)
 December 10, 1941: U.S. declared a war against Germany
 February 27, 1942 (during the japanese occupation): the building
and insured merchandise were burned
 their claimed from Filipinas and the salvage goods were auctioned
for P92,650 who refused since Christen was organized under the
Philippine laws, it was under American jurisdiction which is an
enemy of the Germans
 April 9, 1943: The Director of Bureau of Financing ordered Filipinas
to pay the P92,650 to Christen and it did.
 Filipinas filed with the CFI the P92,650 paid to Christern
 CA affirmed CFI: dismissed the action
 Filed a petition for certiorari
ISSUE: W/N Christern is a public enemy and therefore ceased to be
insured
HELD: YES. Ordered to pay Filipinas P77,208.33, Philippine currency,
less the amount of the premium, in Philippine currency, that should be
returned by the Filipinas for the unexpired term of the policy in
question, beginning December 11, 1941
 Philippine Insurance Law (Act No. 2427, as amended,) in section 8,
provides that "anyone except a public enemy may be insured
 Effect of war, generally. — All intercourse between citizens of
belligerent powers which is inconsistent with a state of war is
prohibited by the law of nations. Such prohibition includes all
negotiations, commerce, or trading with the enemy; all acts which
will increase, or tend to increase, its income or resources; all acts
of voluntary submission to it; or receiving its protection; also all
acts concerning the transmission of money or goods; and all
contracts relating thereto are thereby nullified. It further prohibits
insurance upon trade with or by the enemy, upon the life or lives of
aliens engaged in service with the enemy; this for the reason that
the subjects of one country cannot be permitted to lend their
assistance to protect by insurance the commerce or property of
belligerent, alien subjects, or to do anything detrimental too their
country's interest. The purpose of war is to cripple the power and
exhaust the resources of the enemy, and it is inconsistent that one
country should destroy its enemy's property and repay in insurance
the value of what has been so destroyed, or that it should in such
manner increase the resources of the enemy, or render it aid, and
the commencement of war determines, for like reasons, all trading
intercourse with the enemy, which prior thereto may have been
lawful. All individuals therefore, who compose the belligerent
powers, exist, as to each other, in a state of utter exclusion, and
are public enemies
 In the case of an ordinary fire policy, which grants insurance only
from year, or for some other specified term it is plain that when the
parties become alien enemies, the contractual tie is broken and the
contractual rights of the parties, so far as not vested.
 However, elementary rules of justice (in the absence of specific
provision in the Insurance Law) require that the premium paid by
the respondent for the period covered by its policy from December
11, 1941, should be returned by the petitioner

San Miguel Brewery v. Law Union Rock Insurance Company -


Insurance Proceeds

Facts:

> On Jan. 12, 1918, Dunn mortgaged a parcel of land to SMB to


secure a debt of 10T.

> Mortgage contract stated that Dunn was to have the property
insured at his own expense, authorizing SMB to choose the insurers
and to receive the proceeds thereof and retain so much of the
proceeds as would cover the mortgage debt.

> Dunn likewise authorized SMB to take out the insurance policy for
him.

> Brias, SMB’s general manager, approached Law Union for insurance
to the extent of 15T upon the property. In the application, Brias
stated that SMB’s interest in the property was merely that of a
mortgagee.

> Law Union, not wanting to issue a policy for the entire amount,
issued one for P7,500 and procured another policy of equal amount
from Filipinas Cia de Seguros. Both policies were issued in the name
of SMB only and contained no reference to any other interests in the
propty. Both policies required assignments to be approved and noted
on the policy.
> Premiums were paid by SMB and charged to Dunn. A year later, the
policies were renewed.

> In 1917, Dunn sold the property to Harding, but no assignment of


the policies was made to the latter.

> Property was destroyed by fire. SMB filed an action in court to


recover on the policies. Harding was made a defendant because by
virtue of the sale, he became the owner of the property, although the
policies were issued in SMB’s name.

> SMB sought to recover the proceeds to the extent of its mortgage
credit with the balance to go to Harding.

> Insurance Companies contended that they were not liable to


Harding because their liability under the policies was limited to the
insurable interests of SMB only.

> SMB eventually reached a settlement with the insurance companies


and was paid the balance of it’s mortgage credit. Harding was left to
fend for himself. Trial court ruled against Harding. Hence the appeal.

Issue:

Whether or not the insurance companies are liable to Harding for the
balance of the proceeds of the 2 policies.

Held:NO

Under the Insurance Act, the measure of insurable interest in the


property is the extent to which the insured might be daminified by the
loss or injury thereof. Also it is provided in the IA that the insurance
shall be applied exclusively to the proper interest of the person in
whose name it is made. Undoubtedly, SMB as the mortgagee of the
property, had an insurable interest therein; but it could NOT, an any
event, recover upon the two policies an amount in excess of its
mortgage credit.

By virtue of the Insurance Act, neither Dunn nor Harding could have
recovered from the two policies. With respect to Harding, when he
acquired the property, no change or assignment of the policies had
been undertaken. The policies might have been worded differently so
as to protect the owner, but this was not done.

If the wording had been: “Payable to SMB, mortgagee, as its interests


may appear, remainder to whomsoever, during the continuance of the
risk, may become owner of the interest insured”, it would have proved
an intention to insure the entire interest in the property, NOT merely
SMB’s and would have shown to whom the money, in case of loss,
should be paid. Unfortunately, this was not what was stated in the
policies.

If during the negotiation for the policies, the parties had agreed that
even the owner’s interest would be covered by the policies, and the
policies had inadvertently been written in the form in which they were
eventually issued, the lower court would have been able to order that
the contract be reformed to give effect to them in the sense that the
parties intended to be bound. However, there is no clear and
satisfactory proof that the policies failed to reflect the real agreement
between the parties that would justify the reformation of these two
contracts.

San Miguel Brewery V. Law Union And Rock Insurance Co. (1920)
Lessons Applicable:

 Mortgagor (Insurance)
 Measure of Insurable Interest (Insurance)
 Effect of Change of Interest in Thing Insured (Insurance)
 Effect of transfer of thing insured (Insurance)
Laws Applicable: sec. 16,sec. 19 (now sec. 20),sec. 50,sec.55 (now
sec. 58) of the Insurance Code (all old law)
FACTS:

 In the contract of mortgage, the owner P.D. Dunn had agreed, at


his own expense, to insure the mortgaged property for its full value
and to indorse the policies in such manner as to authorize the
Brewery Company to receive the proceeds in case of loss and to
retain such part thereof as might be necessary to satisfy the
remainder then due upon the mortgage debt. Instead, however, of
effecting the insurance himself Dunn authorized and requested the
Brewery Company to procure insurance on the property in the
amount of P15,000 at Dunn's expense.
 San Miguel insured the property only as mortgagee.
 Dunn sold the propert to Henry Harding. The insurance was not
assigned by Dunn to Harding.
 When it was destroyed by fire, the two companies settled with San
Miguelto the extent of the mortgage credit.
 RTC: Absolved the 2 companies from the difference. Henry
Harding is not entitled to the difference between the mortgage
credit and the face value of the policies.
 Henry Harding appealed.
ISSUE:
1. W/N San Miguel has insurable interest as mortgagor only to the
extent of the mortgage credit - YES
2. W/N Harding has insurable interest as owner - NO

HELD: affirmed
 section 19 of the Insurance Act:
 a change of interest in any part of a thing insured unaccompanied
by a corresponding change of interest in the insurance, suspends
the insurance to an equivalent extent, until the interest in the thing
and the interest in the insurance are vested in the same person
 section 55:
 the mere transfer of a thing insured does not transfer the policy,
but suspends it until the same person becomes the owner of both
the policy and the thing insured
 Undoubtedly these policies of insurance might have been so framed
as to have been "payable to the San Miguel Brewery, mortgagee,
as its interest may appear, remainder to whomsoever, during the
continuance of the risk, may become the owner of the interest
insured." (Sec 54, Act No. 2427.) Such a clause would have proved
an intention to insure the entire interest in the property, not merely
the insurable interest of the San Miguel Brewery, and would have
shown exactly to whom the money, in case of loss, should be paid.
But the policies are not so written.
 The blame for the situation thus created rests, however, with the
Brewery rather than with the insurance companies, and there is
nothing in the record to indicate that the insurance companies were
requested to write insurance upon the insurable interest of the
owner or intended to make themselves liable to that extent
 If by inadvertence, accident, or mistake the terms of the contract
were not fully set forth in the policy, the parties are entitled to
have it reformed. But to justify the reformation of a contract, the
proof must be of the most satisfactory character, and it must
clearly appear that the contract failed to express the real
agreement between the parties
 In the case now before us the proof is entirely insufficient to
authorize reformation.

Saura Import Export Co. v. Philippine International Surety -


Cancellation of Policy

Facts:

> On Dec. 26, 1952, Saura mortgaged to PNB its registered parcel of
land in Davao to secure the payment of a promissory note of P27T.

> A building of strong materials which was also owned by Saura, was
erected on the parcel of land and the building had always been
covered by insurance even before the execution of the mortgage
contract.

> Pursuant to the mortgage agreement which required Saura to


insure the building and its contents, it obtained a fire insurance for
P29T from PISC for a period of 1 year starting Oct. 2, 1954.
> The mortgage also required Saura to endorse the insurance policy
to PNB. The memo stated: Loss if any, payable to PNG as their
interest may appear, subject to the terms, conditions and warranties
of this policy.

> The policy was delivered to PNB by Saura.

> On Oct. 15, 1954, barely 13 days after the issuance of the fire
insurance, PISC canceled the same, effective as of the date of
issue. Notice of the cancellation was sent to PNB in writing and was
received by the bank on Nov. 8, 1954.

> On Apr. 6, 1955, the building and its contents worth P4,685 were
burned. On April 11, 1985, Saura filed a claim with PISC and
mortgagee bank.

> Upon presentation of notice of loss with PNB, Saura learned for the
first time that the policy had been previously canceled by PISC, when
Saura’s folder in the bank’s file was opened and the notice of the
cancellation by PISC was found.

Issue:

Whether or not there was proper cancellation of the policy?

Held: NO.

The policy in question does NOT provide for the notice of cancellation,
its form or period. The Insurance Law does not likewise provide for
such notice. This being the case, it devolves upon the Court to apply
the generally accepted principles of insurance, regarding cancellation
of the insurance policy by the insurer.

Actual notice of cancellation in a clear and unequivocal manner,


preferably in writing should be given by the insurer to the insuredso
that the latter might be given an opportunity to obtain other insurance
for his own protection. The notice should be personal to the insurer
and not to and/or through any unauthorized person by the
policy. Both the PSIC and the PNB failed, wittingly or unwittingly to
notify Saura of the cancellation made.
The insurer contends that it gave notice to PNB as mortgagee of the
property and that was already substantial compliance with its duty to
notify the insured of the cancellation of the policy. But notice to the
bank, as far as Saura herein is concerned, is not effective notice. PISC
is then ordered to pay Saura P29T, the amount involved in the policy
subject matter of this case

Saura Import & Export Co., Inc. V. Philippine International Surety Co.,
Inc. (1963)

Lessons Applicable: Mortgagor (Insurance)


Laws Applicable:
FACTS:

 Saura Import & Export Co Inc., mortgaged to the Phil. National


Bank, a parcel of land.
 The mortgage was amended to guarantee an increased amount,
bringing the total mortgaged debt to P37,000
 On the land mortgage is a building owned by Saura Import &
Export Co Inc. which was insured with Philippine International
Surety (Insurer) even before the mortgage contract so it was
required to endorse to mortgagee PNB
 October 15, 1954: Barely 13 days after the issuance of the fire
insurance policy, the insurer cancelled it. Notice of the cancellation
was given to PNB (mortgagee). But Saura (insured) was not
informed.
 April 6, 1955: The building and all its contents worth P40,685.69
were burned so Saura filed a claim with the Insurer and mortgagee
Bank
 RTC: dismissed
ISSUE: W/N Philippine International Surety should be held liable for
the claim because notice to only the mortgagee is not substantial
HELD:YES. Appealed from is hereby reversed. Philippine International
Surety Co., Inc., to pay Saura Import & Export Co., Inc., P29,000
 It was the primary duty of Philippine International Surety to notify
the insured, but it did not
 If a mortgage or lien exists against the property insured, and the
policy contains a clause stating that loss, if any, shall be payable to
such mortgagee or the holder of such lien as interest may appear,
notice of cancellation to the mortgagee or lienholder alone is
ineffective as a cancellation of the policy to the owner of the
property.
 liability attached principally the insurance company, for its failure
to give notice of the cancellation of the policy to Saura
 it is unnecessary to discuss the errors assigned against appellee
bank

Palilieo v. Cosio - Insurance Proceeds

Facts:

> On Dec. 18, 1951, Palileo obtained from Cosio a loan of P12T.

> To secure payment, Cosio required Palileo to sign a document


known as “conditional sale of residential building”, purporting to
convey to Cosio, with a right to repurchase (on the part of Palileo), a
two-story building of strong materials belonging to Palileo.

> After execution of the document, Cosio insured the building against
fire with Associated Insurance & Surety Co. (Associated) for 15T.

> The insurance policy was issued in the name of Cosio.

> The building was partly destroyed by fire and after proper demand,
Cosio was able to collect from the insurance company an indemnity of
P13,107.

> Palileo demanded from Cosio that she be credited with the
necessary amount to pay her obligation out of the insurance proceeds,
but Cosio refused to do so.
> Trial Court found that the debt had an unpaid balance of P12T. It
declared the obligation of Palileo to Cosio fully compensated by virtue
of the proceeds collected by Cosio and further held that the excess of
P1,107 (13,107 – 12,000) be refunded to Palileo

Issue:

Whether or not the trial court was justified in considering the


obligation of Palileo fully compensated by the insurance amount that
Cosio was able to collect from Associated, and whether or not the trial
court was correct in requiring Cosio to refund the excess of P1,107 to
Palileo.

Held:

NO

The rule is that “where a mortgagee, independently of the mortgagor,


insures the mortgaged property in his own name and for his own
interest, he is entitled to the insurance proceeds in case of loss, but in
such case, he is not allowed to retain his claim against the mortgagor,
but is passed by subrogation to the insurer to the extent of the money
paid.”

The lower court erred in declaring that the proceeds of the insurance
taken out by Cosio on the property insured to the benefit of Palileo and
in ordering the former to deliver to the latter, the difference between
the indebtedness and the amount of insurance received by Cosio. In
the light of this ruling, the correct solution would be that the proceeds
of the Insurance be delivered to Cosio, but her claim against Palileo
should be considered assigned to the insurance company who is
deemed subrogated to the rights of Cosio to the extent of the money
paid as indemnity.

Insurance Case Digest: Palileo V. Cosio (1955)


Lessons Applicable: Mortgagor (Insurance)

Laws Applicable:
FACTS:
 Cherie Palileo (debtor-mortgagor) filed a complaint against Beatriz
Cosio (creditor-mortgagee) praying that their transaction be one of
a loan with an equitable mortgage to secure the payment of the
loan. The original counsel of Cosio Atty. Guerrero being appointed
Undersecretary of Foreign Affairs so she forgot the date of the trial
and she was substituted.
 it is a loan of P12,000 secured by a "Conditional Sale of Residential
Building" with right to repurchase. After the execution of the
contract, Cosio insured in her name the building with Associated
Insurance & Surety Co. against fire.
 The building was partly destroyed by fire so she claimed an
indemnity of P13,107
 Palileo demanded that the amount of insurance proceeds be
credited to her loan
 RTC: it is a loan with equitable mortgage so the insurance proceeds
should be credited to the loan and refund the overpayment.
ISSUE: W/N Cosio as mortgagee is entitled to the insurance proceeds
for her own benefit
HELD: YES. Modify. collection of insurance proceeds shall not be
deemed to have compensated the obligation of the Palileo to Cosio,
but bars the Cosio from claiming its payment from the Palileo; and
Cosio shall pay to Palileo P810 representing the overpayment made by
Palileo by way of interest on the loan.
 When the the mortgagee may insure his interest in the property
independently of the mortgagor , upon the destruction of the
property the insurance money paid to the mortgagee will not inure
to the benefit of the mortgagor, and the amount due under the
mortgage debt remains unchanged. The mortgagee, however, is
not allowed to retain his claim against the mortgagor, but it passes
by subrogation to the insurer, to the extent of the insurance money
paid
 It is true that there are authorities which hold that "If a mortgagee
procures insurance on his separate interest at his own expense and
for his own benefit, without any agreement with the mortgagor
with respect thereto, the mortgagor has no interest in the policy,
and is not entitled to have the insurance proceeds applied in
reduction of the mortgage debt" But these authorities merely
represent the minority view

Grepalife v. CA - Real Party In Interest

Facts:

> A contract of group life insurance was executed between Grepalife


and DBP. Grepalife agreed to insure the lives of eligible housing loan
mortgagors of DBP.

> Dr. Wilfredo Leuterio, a physician and a housing debtor of DBP


applied for membership in the group life insurance plan.

> In an application form, Dr. Leuterio answered questions concerning


his health stating that he is in good health and has never consulted a
physician for or a heart condition, high blood pressure, cancer,
diabetes, lung, kidney or stomach disorder or any other physical
impairment.

> Grepalife issued the insurance coverage of Dr. Leuterio, to the


extent of his DBP mortgage indebtedness amounting to eighty-six
thousand, two hundred (P86,200.00) pesos.

> Dr. Leuterio died due to "massive cerebral hemorrhage."


Consequently, DBP submitted a death claim to Grepalife.

> Grepalife denied the claim alleging that Dr. Leuterio was not
physically healthy when he applied for an insurance coverage and
insisted that Dr. Leuterio did not disclose that he had been suffering
from hypertension, which caused his death. Allegedly, such non-
disclosure constituted concealment that justified the denial of the
claim.
> The widow of the late Dr. Leuterio, filed a complaint against
Grepalife for "Specific Performance with Damages." During the trial,
Dr. Hernando Mejia, who issued the death certificate, was called to
testify. Dr. Mejia’s findings, based partly from the information given by
the widow, stated that Dr. Leuterio complained of headaches
presumably due to high blood pressure. The inference was not
conclusive because Dr. Leuterio was not autopsied, hence, other
causes were not ruled out.

> RTC ruled in favor of widow and against Grepalife. Grepalife


appealed contending that the wife was not the proper party in interest
to file the suit, since it is DBP who insured the life of Dr. Leuterio.

Issue:

Whether or not the widow is the real party in interest, (not DBP) and
has legal standing to file the suit.

Held: YES.

Grepalife alleges that the complaint was instituted by the widow of Dr.
Leuterio, not the real party in interest, hence the trial court acquired
no jurisdiction over the case. It argues that when the Court of Appeals
affirmed the trial court’s judgment, Grepalife was held liable to pay the
proceeds of insurance contract in favor of DBP, the indispensable party
who was not joined in the suit.

To resolve the issue, we must consider the insurable interest in


mortgaged properties and the parties to this type of contract. The
rationale of a group insurance policy of mortgagors, otherwise known
as the "mortgage redemption insurance," is a device for the protection
of both the mortgagee and the mortgagor. On the part of the
mortgagee, it has to enter into such form of contract so that in the
event of the unexpected demise of the mortgagor during the
subsistence of the mortgage contract, the proceeds from such
insurance will be applied to the payment of the mortgage debt,
thereby relieving the heirs of the mortgagor from paying the
obligation.
In a similar vein, ample protection is given to the mortgagor under
such a concept so that in the event of death; the mortgage obligation
will be extinguished by the application of the insurance proceeds to the
mortgage indebtedness. Consequently, where the mortgagor pays the
insurance premium under the group insurance policy, making the loss
payable to the mortgagee, the insurance is on the mortgagor’s
interest, and the mortgagor continues to be a party to the contract. In
this type of policy insurance, the mortgagee is simply an appointee of
the insurance fund, such loss-payable clause does not make the
mortgagee a party to the contract.

The insured private respondent did not cede to the mortgagee all his
rights or interests in the insurance, the policy stating that: "In the
event of the debtor’s death before his indebtedness with the Creditor
[DBP] shall have been fully paid, an amount to pay the outstanding
indebtedness shall first be paid to the creditor and the balance of sum
assured, if there is any, shall then be paid to the beneficiary/ies
designated by the debtor." When DBP submitted the insurance claim
against petitioner, the latter denied payment thereof, interposing the
defense of concealment committed by the insured. Thereafter, DBP
collected the debt from the mortgagor and took the necessary action
of foreclosure on the residential lot of private respondent

And since a policy of insurance upon life or health may pass by


transfer, will or succession to any person, whether he has an insurable
interest or not, and such person may recover it whatever the insured
might have recovered, 14 the widow of the decedent Dr. Leuterio may
file the suit against the insurer, Grepalife.

As to the question of whether there was concealment, CA held as


affirmed by the SC that contrary to Grepalife’s allegations, there was
no sufficient proof that the insured had suffered from hypertension.
Aside from the statement of the insured’s widow who was not even
sure if the medicines taken by Dr. Leuterio were for hypertension, the
appellant had not proven nor produced any witness who could attest to
Dr. Leuterio’s medical history.

The fraudulent intent on the part of the insured must be established to


entitle the insurer to rescind the contract. Misrepresentation as a
defense of the insurer to avoid liability is an affirmative defense and
the duty to establish such defense by satisfactory and convincing
evidence rests upon the insurer. In the case at bar, the petitioner
failed to clearly and satisfactorily establish its defense, and is therefore
liable to pay the proceeds of the insurance

Great Pacific Life Assurance Corp. V. CA (1999)


Lessons Applicable:

 Credit in Life and Health Insurance (Insurance)


 Mortgagor (Insurance)
Laws Applicable: Sec. 8 of Insurance Code
FACTS:
 A contract of group life insurance was executed between Great
Pacific Life Assurance Corporation Grepalife) and Development
Bank of the Philippines (DBP)
 Grepalife agreed to insure the lives of eligible housing loan
mortgagors of DBP
 November 11, 1983: Dr. Wilfredo Leuterio, a physician and a
housing debtor of DBP applied for membership in the group life
insurance plan
 Dr. Leuterio answered questions concerning his health condition as
follows:

“7. Have you ever had, or consulted, a physician for a heart


condition, high blood pressure, cancer, diabetes, lung, kidney or
stomach disorder or any other physical impairment?

Answer: No. If so give details ___________.

8. Are you now, to the best of your knowledge, in good health?

Answer: [ x ] Yes [ ] No.”[4]


 November 15, 1983: Grepalife issued Certificate No. B-18558, as
insurance coverage of Dr. Leuterio, to the extent of his DBP
mortgage indebtedness amounting to P86,200
 August 6, 1984: Dr. Leuterio died due to “massive cerebral
hemorrhage.”
 DBP submitted a death claim to Grepalife
 Grepalife denied the claim alleging that Dr. Leuterio was not
physically healthy when he applied
 RTC: Favored Medarda V. Leuterio (widow) and held Grepalife
(insurer) liable to pay DBP (creditor of the insured Dr. Wilfredo
Leuterio)
 CA sustained
ISSUE:
1. W/N DBP has insurable interest as creditor - YES
2. W/N Grepalife should be held liable - YES

HELD:
1. YES
 In this type of policy insurance, the mortgagee is simply an
appointee of the insurance fund, such loss-payable clause does not
make the mortgagee a party to the contract
 Section 8 of the Insurance Code provides:

“Unless the policy provides, where a mortgagor of property effects


insurance in his own name providing that the loss shall be payable to
the mortgagee, or assigns a policy of insurance to a mortgagee, the
insurance is deemed to be upon the interest of the mortgagor, who
does not cease to be a party to the original contract, and any act of
his, prior to the loss, which would otherwise avoid the insurance, will
have the same effect, although the property is in the hands of the
mortgagee, but any act which, under the contract of insurance, is to
be performed by the mortgagor, may be performed by the mortgagee
therein named, with the same effect as if it had been performed by the
mortgagor.”
 The insured Dr. Wilfredo Leuterio did not cede to the mortgagee all
his rights or interests in the insurance. When Grepalife denied
payment, DBP collected the debt from the mortgagor and took the
necessary action of foreclosure on the residential lot of Dr. Wilfredo
Leuterio
 Insured may be regarded as the real party in interest, although he
has assigned the policy for the purpose of collection, or has
assigned as collateral security any judgment he may obtain
2. YES
 medical findings were not conclusive because Dr. Mejia did not
conduct an autopsy
 widow who was not even sure if the medicines taken by Dr.
Leuterio were for hypertension
 Grepalife failed to establish that there was concealment made by
the insured, hence, it cannot refuse payment of the claim
 fraudulent intent on the part of the insured must be established to
entitle the insurer to rescind the contract. Misrepresentation as a
defense of the insurer to avoid liability is an affirmative defense
and the duty to establish such defense by satisfactory and
convincing evidence rests upon the insurer
 The policy states that upon receipt of due proof of the Debtor’s
death during the terms of this insurance, a death benefit in the
amount of P86,200.00 shall be paid. In the event of the debtor’s
death before his indebtedness with the creditor shall have been
fully paid, an amount to pay the outstanding indebtedness shall
first be paid to the Creditor and the balance of the Sum Assured, if
there is any shall then be paid to the beneficiary/ies designated by
the debtor.
 DBP foreclosed in 1995 their residential lot, in satisfaction of
mortgagor’s outstanding loan
 insurance proceeds shall inure to the benefit of the heirs of the
deceased person or his beneficiaries
 Equity dictates that DBP should not unjustly enrich itself at the
expense of another (Nemo cum alterius detrimenio
protest). Hence, it cannot collect the insurance proceeds, after it
already foreclosed on the mortgage

PARTIES TO AN INSURANCE CONTRACT

Nario v. Philamlife Insurance Company - Loan Application and


Surrender of Policy

Facts:

> Mrs. Nario applied for and was issued a life Insurance policy (no.
503617) by PHILAMLIFE under a 20-yr endowment plant, with a face
value of 5T. Her husband Delfin and their unemancipated son Ernesto
were her revocable beneficiaries.

> Mrs. Nario then applied for a loan on the above policy with
PHILAMLIFE w/c she is entitled to as policy holder, after the policy has
been in force for 3 years. The purpose of such loan was for the school
expenses of Ernesto.

> The application bore the written signature and consent of Delfin in 2
capacities

o As one of the irrevocable beneficiaries of the policy

o As father-guardian of Ernesto and also the legal administrator of


the minor’s properties pursuant to Art. 320 of the CC.

> PHILAMLIFE denied the loan application contending that written


consent of the minor son must not only be given by his father as legal
guardian but it must also be authorized by the court in a competent
guardianship proceeding.
> Mrs. Nario then signified her decision to surrender her policy and
demand its cash value which then amounted to P 520.

> PHILAMLIFE also denied the surrender of the policy on the same
ground as that given in disapproving the loan application.

> Mrs. Nario sued PHILAMLIFE praying that the latter grant their loan
application and/or accept the surrender of said policy in exchange for
its cash value.

> PHILAMLIFE contends that the loan application and the surrender of
the policy involved acts of disposition and alienation of the property
rights of the minor, said acts are not within the power of administrator
granted under Art. 320 in relation to art. 326 CC, hence court
authority is required.

Issue:

Whether or not PHILAMLIFE was justified in refusing to grant the loan


application and the surrender of the policy.

Held: YES.

SC agreed with the trial court that the vested interest or right of the
beneficiaries in the policy should be measured on its full face value and
not on its cash surrender value, for in case of death of the insured,
said beneficiaries are paid on the basis of its face value and in case the
insured should discontinue paying premiums, the beneficiaries may
continue paying it and are entitled to automatic extended term or
paid-up insurance options and that said vested right under the policy
cannot be divisible at any given time.

SC also agreed with TC that the said acts (loan app and surrender)
constitute acts of disposition or alienation of property rights and not
merely management or administration because they involve the
incurring or termination of contractual obligations.

Under the laws (CC and rules of Court) The father is constituted as the
minor’s legal administrator of the propty, and when the propty of the
child is worth more than P2T (as in the case at bar, the minor’s propty
was worth 2,500 his ½ share as beneficiary), the father a must file a
petition for guardianship and post a guardianship bond. In the case at
bar, the father did not file any petition for guardianship nor post a
guardianship bond, and as such cannot possibly exercise the powers
vested on him as legal administrator of the minor’s property. The
consent give for and in behalf of the son without prior court
authorization to the loan application and the surrender was insufficient
and ineffective and PHILAMLIFE was justified in disapproving the said
applications.

Assuming that the propty of the ward was less than 2T, the effect
would be the same, since the parents would only be exempted from
filing a bond and judicial authorization, but their acts as legal
administrators are only limited to acts of management or
administration and not to acts of encumbrance or disposition.

Nario V. Philippine American Life Insurance Co. Of Canada (1967)

Lessons Applicable: Irrevocable Designation (Insurance)

FACTS:
 June 12, 1959: Philippine American Life Insurance Co. issued a life
insurance to Mrs. Alejandra Santos-Mario a life insurance policy
under a 20-year endowment plan, with a face value of
P5,000 designating her husband Delfin Nario and their
unemancipation son Ernesto Nario, as her irrevocable beneficiaries
 June, 1963: She submitted her loan application to the life insurance
co. with signature of her husband in two capacities:
 irrevocable beneficiaries
 father-guardian of minor irrevocable beneficiary Ernesto
 Insurance Co. denied asking that the legal guardian must
be authorized by the court in a competent guardianship proceeding
 Upon denial, she opted to surrender her insurance policy in
exchange of its cash surrender value of P520 but it was also denied
on the same ground
 September 10, 1963: Mrs. Alejandra Santos-Nario and her
husband, Delfin Nario, brought suit against the Philippine American
Life Insurance Co
 RTC: favored the insurance company
 CA: vested interest or right of the beneficiaries in the policy should
be measured on its full face value and not on its cash surrender
value, for in case of death of the insured, said beneficiaries are
paid on the basis of its face value and in case the insured should
discontinue paying premiums, the beneficiaries may continue
paying it and are entitled to automatic extended term or paid-up
insurance options, etc. and that said vested right under the policy
cannot be divisible at any given time. policy loan and surrender of
policy constitute acts of disposition or alienation of property rights
and not merely of management or administration because they
involve the incurring or termination of contractual obligations
ISSUE: W/N parents as guardians can enter into transactions for the
benefit of minor irrevocable beneficiaries.
HELD: NO. Affirmed.
 SEC. 7. Parents as guardians. — When the property of the child
under parental authority is worth two thousand pesos or less, the
father or the mother, without the necessity of court appointment,
shall be his legal guardian. When the property of the child is worth
more than two thousand pesos, the father or the mother shall be
considered guardian of the child's property, with the duties and
obligations of guardians under these rules, and shall file the
petition required by Section 2 hereof. For good reasons the court
may, however, appoint another suitable person.
 even if worth less than P2,000 parent's authority over the estate of
the ward as a legal-guardian would not extend to acts of
encumbrance or disposition, as distinguished from acts of
management or administration.
Villanueva v. Oro - Insurance Proceeds

Facts:

> West Coast Life Insurance Company issued two policies of insurance
on the life of Esperanza Villanueva, one for 2T, maturing April 1, 1943;
and other for 3T maturing Mar. 31, 1943.

> In both policies, West agreed to pay 2T either to Esperanza if still


living on Apr 1, 1943; or to beneficiary Bartolome Villanueva, or the
father of the insured immediately upon receipt of the proof of death of
Esperanza.

> The policy also gave her the right to change the beneficiary.

> In 1940, Bartolome died, and he was substituted as beneficiary


under the policies by Mariano, Esparanza’s brother.

> Esperanza died in 1944 without having collected the insurance


proceeds. Adverse claims for the proceeds were presented by the
estate of Esperanza on one hand and by Mariano on the other.

> CFI held that the estate of Esperanza was entitled to the proceeds
to the exclusion of the beneficiary.

Issue:

Whether or not the beneficiary is entitled to the proceeds.

Held:NO.

Under the policies, the insurer obligated itself to pay the insurance
proceeds to: (1) the insured if the latter lived on the dates of maturity;
or (2) the beneficiary if the insured died during the continuance of the
policies. The first contingency excludes the second, and vice versa. In
other words, as the insured Esperanza was living on April 1 and March
31, 1943, the proceeds are payable exclusively to her or to her estate
unless she had before her death otherwise assigned the matured
policies.

The beneficiary could be entitled to said proceeds only in default of the


first contingency. To sustain the beneficiary’s claim would be to
altogether eliminate from the policies the condition that the insurer
“agrees to pay to the insured if living.”

This conclusion tallies with American Authorities who say that: The
interest of the insured in the proceeds of the insurance depends upon
his survival of the expiration of the endowment period. Upon the
insured’s death, within the period, the beneficiary will take, as against
the personal representatives the endowment period, the benefits are
payable to him or to his assignee, notwithstanding a beneficiary is
designated in the policy. (AmJur and Couch Cyclopedia of Insurance
Law)

Villanueva V. Oro (1948)

Lessons Applicable: Insured Outlives Policy (Insurance)


Laws Applicable:
FACTS:
 West Coast Life Insurance Company issued 2 policies of insurance
on the life of Esperanza J. Villanueva:
 2,000 php - maturing on April 1, 1943
 if living, on the 1st day of April 1943 - to insured
 upon death during the continuance of this policy - to the
beneficiary Bartolome Villanueva, father of the insured, with right
on the part of the insured to change the beneficiary
 1940: Bartolome Villanueva died, Mariano J. Villanueva duly
substituted as beneficiary, a brother of the insured
 3,000 php - maturing on March 31, 1943
 Esperanza J. Villanueva survived the insurance period, for she died
only on October 15, 1944, without, however, collecting the
insurance proceeds.
 CFI: estate of the insured Esperanza is entitled to the insurance
proceeds
ISSUE: W/N the estate of insured Esperanza should be entitled to the
insurance proceeds since she outlived the insurance policy
HELD: YES. appealed order is, therefore, hereby affirmed
 To sustain the beneficiary's claim would be altogether eliminate
from the policies the condition that the insurer "agrees to pay . . .
to the insured hereunder, if living
 Upon the insured's death, within the period, the beneficiary will
take, as against the personal representative or the assignee of the
insured. Upon the other hand, if the insured survives the
endowment period, the benefits are payable to him or to his
assignee, notwithstanding a beneficiary is designated in the policy

Philamlife v. Pineda - Life Insurance

Facts:

> On Jan. 15 1963, Dimayuga processed an ordinary life insurance


policy from Philamlife and designated his wife and children as
irrevocable beneficiaries.

> On Feb. 22, 1980, Dimayuga filed a petition in court to amend the
designation of the beneficiaries in his policy from irrevocable to
revocable.

> Lower Court granted the petition.

Issue:

Whether or not the court erred in granting Dimayuga’s petition.

Held:

YES.

Under the Insurance Act, the beneficiary designated in a life insurance


contract cannot be changed without the consent of the beneficiary
because he has a vested interest in the policy. The policy contract
states that the designation of the beneficiaries is
irrevocable. Therefore, based on the said provision of the contract,
not to mention the law then applicable, it is only with the consent of all
the beneficiaries that any change or amendment in the poicy may be
legally and validly effected. The contract between the parties is the
law binding on them. (This case rule is no longer controlling under the
Insurance Code.)

Philam v Pineda G.R. No. L-54216 July 19, 1989

Facts:
Pineda procured an ordinary life insurance policy from the petitioner
company and designated his wife and children as irrevocable
beneficiaries.
He then filed a petition to amend the designation of the beneficiaries in
his life policy from irrevocable to revocable.
The judge granted the request.
Petitioner promptly filed a motion but was denied. Hence, this petition.

Issues:
1. WON the designation of the irrevocable beneficiaries could be
changed or amended without the consent of all the irrevocable
beneficiaries.
2. WON the irrevocable minor beneficiaries could give consent to the
change in designation

Held: No to both. Petition dismissed.

Ratio:
Under the Insurance Act, the beneficiary designated in a life insurance
contract cannot be changed without the consent of the beneficiary
because he has a vested interest in the policy.
There was an express stipulation to this effect: “It is hereby understood
and agreed that, notwithstanding the provisions of this policy to the
contrary, inasmuch as the designation of the primary/contingent
beneficiary/beneficiaries in this Policy has been made without reserving
the right to change said beneficiary/ beneficiaries, such designation may
not be surrendered to the Company, released or assigned; and no right
or privilege under the Policy may be exercised, or agreement made with
the Company to any change in or amendment to the Policy, without the
consent of the said beneficiary/beneficiaries.”
The alleged acquiescence of the six (6) children beneficiaries of the
policy cannot be considered an effective ratification due to the fact that
they were minors. Neither could they act through their father insured
since their interests are quite divergent from one another.
Therefore, the parent-insured cannot exercise rights and/or privileges
pertaining to the insurance contract, for otherwise, the vested rights of
the irrevocable beneficiaries would be rendered inconsequential.
Of equal importance is the well-settled rule that the contract between
the parties is the law binding on both of them and for so many times,
this court has consistently issued pronouncements upholding the validity
and effectivity of contracts. Likewise, contracts which are the private
laws of the contracting parties should be fulfilled according to the literal
sense of their stipulations, for contracts are obligatory, no matter in
what form they may be, whenever the essential requisites for their
validity are present
The change in the designation of was not within the contemplation of
the parties. The lower court instead made a new contract for them. It
acted in excess of its authority when it did so.

Insular Life vs. Ebrado

Facts:

> Buenaventura Ebrado was issued al life plan by Insular


Company. He designated Capriona as his beneficiary, referring to her
as his wife.

> The insured then died and Carponia tried to claim the proceeds of
the said plan.

> She admitted to being only the common law wife of the insured.

> Pascuala, the legal wife, also filed a claim asserting her right as the
legal wife. The company then filed an action for interpleader.

Issue:

Whether or not the common law wife named as beneficiary can collect
the proceeds.

Held:NO.

The civil code prohibitions on donations made between persons guilty


of adulterous concubinage applies to insurance contracts. On matters
not specifically provided for by the Insurance Law, the general rules on
Civil law shall apply. A life insurance policy is no different from a civil
donation as far as the beneficiary is concerned, since both are founded
on liberality.

Why was the common law wife not ed to collect the proceeds despite
the fact that she was the beneficiary? Isn’t this against Sec. 53?

It is true that SC went against Sec. 53. However, Sec. 53 is NOT the
only provision that the SC had to consider. Art. 739 and 2012 of CC
prohibit persons who are guilty of adultery or concubinage from being
beneficiaries of the life insurance policies of the persons with whom
they committed adultery or concubinage. If the SC used only Sec. 53,
it would have gone against Art. 739 and 2012.

The Insular Life Assurance Co. Ltd. V. Ebrado (1977)

Lessons Applicable:

 Art. 2011 Civil Code (Insurance)


 Invalid Designation (Insurance)
FACTS:
 September 1, 1968: Buenaventura Cristor Ebrado was issued by
The Insular Life Assurance Co., Ltd., Policy on a whole-life for
P5,882.00 with a, rider for Accidental Death and designated
Carponia T. Ebrado as the revocable beneficiary in his policy
 October 21, 1969: Buenaventura was hit by a falling branch and
died.
 Carponia filed a claim as the designated beneficiary, although she
admits that they were merely living as husband and wife without
the benefit of marriage
 Pascuala Vda. de Ebrado also filed her claim as the widow of the
deceased insured
 In doubt as to whom the insurance proceeds shall be paid, the
insurer, The Insular Life Assurance Co., Ltd. commenced an action
for Interpleader bef. the CFI
 CFI: Carponia was disqualified because of adultery
 CA: affirmed CFI decision
ISSUE: W/N Carponia is disqualified for violating the Civil Code which
supplements the silent Insurance Code.
HELD: YES. CA affirmed.

Civil Code
Art. 2011
Art. 2011. The contract of insurance is governed by special laws.
Matters not expressly provided for in such special laws shall be
regulated by this Code.
Art. 2012
Art. 2012. Any person who is forbidden from receiving any donation
under Article 739 cannot be named beneficiary of a life insurance
policy by the person who cannot make any donation to him,
according to said article.
Art. 739
Art. 739. The following donations shall be void:

(1) Those made between persons who were guilty of adultery or


concubinage at the time of the donation;
(2) Those made between persons found guilty of the same criminal
offense, in consideration thereof;
(3) Those made to a public officer or his wife, descedants and
ascendants, by reason of his office.

In the case referred to in No. 1, the action for declaration of nullity


may be brought by the spouse of the donor or donee; and the guilt
of the donor and donee may be proved by preponderance of
evidence in the same action.
 Common-law spouses are, definitely, barred from receiving
donations from each other
 In essence, a life insurance policy is no different from a civil
donation insofar as the beneficiary is concerned. Both are founded
upon the same consideration: liberality. A beneficiary is like a
donee, because from the premiums of the policy which the insured
pays out of liberality, the beneficiary will receive the proceeds or
profits of said insurance. As a consequence, the proscription in
Article 739 of the new Civil Code should equally operate in life
insurance contracts.
 We do not think that a conviction for adultery or concubinage is
exacted before the disabilities mentioned in Article 739 may
effectuate.
 requisite proof of common-law relationship between the insured
and the beneficiary has been conveniently supplied by the
stipulations between the parties in the pre-trial conference of the
case

Southern Luzon Employee’s Association v. Golpeo - Insurance


Beneficiaries

Facts:

> SLEA is composed of laborers and employees of the LTBC and BTC
(now BLTB Co.), and one of its purposes is mutual aid of its members
and their dependents in case of death.

> Roman Concepcion was a member until his death in 1950.

> In 1949, SLEA adopted a resolution providing that: A member


may, if he chooses, put down his common law wife and/or children he
had with her as his beneficiaries; and such person so named by the
member will be the sole persons to be recognized by SLEA regarding
claims for condolence contributions.

> Roman listed as his beneficiaries Aquilina Maloles and their 4


children. After his death, SLEA was able to collect voluntary
contribution from its members amounting to P2,205.

> Three sets of claimants to the amount presented themselves to the


association namely:

o Juanita Golpeo, legal wife, and her children

o Aquilina Maloles, the common law wife, and her children


o Elsie Hicban, another common law wife of Roman, and her child.

> SLEA then filed an action for interpleader against the 3 conflicting
claimants.

> Trial court rendered a decision declaring Maloles and her children
the sole beneficiaries of the amount citing Del Val v. Del Val.

> Only Golpeo appealed. She argues that:

> The insurance code does not apply since the association is not an
insurance company but a mutual benefit association.

> The stipulation between SLEA and Roman was void for being
contrary to law, public morals and public policy, pursuant to Art. 739
of the CC ( donations between persons guilty of concubinage at the
time of donation are void)

Issue:

Whether or not Golpeo, the legal wife is entitled to the amount.

Held:NO.

First of all, the lower court did not consider the association as a regular
insurance company, but merely ruled that the death benefit in
question is analogous to insurance. Besides, even the Administrative
Code describes a mutual benefit company as one which provides any
method of life insurance among its members out of dues or
assessments collected from its membership.

Secondly, without considering the intimation in the brief for Maloles


that Golpeo, by her silence and actions had acquiesced in the illicit
relations between her husband and Maloles, Golpeo’s argument would
certainly NOT apply to the children of Maloles likewise named
beneficiaries by the deceased. As a matter of fact, the NCC recognizes
certain successional rights of illegitimate children

Southern Luzon Employees' Ass. V. Golpeo, Et Al. (1954)

Lessons Applicable: Invalid Designation (Insurance)


FACTS:
 Roman A. Concepcion listed as his beneficiaries Aquilina Maloles,
Roman M. Concepcion, Jr., Estela M. Concepcion, Rolando M.
Concepcion and Robin M. Concepcion for the death benefit of
an association amounting to P2,505
 Two sets of claimants presented themselves:
 Juanita Golpeo, legal wife and her children, named beneficiaries by
the deceased
 Marcelino and Josefina Concepcion intervened in their own
right aligning themselves Juanita Golpeo and her minor children
 Elsie Hicban, another common law wife and her child
 RTC:Aquilina Maloles and her children the sole beneficiaries
 Only the Juanita Golpeo and her minor children and the intervenors
Marcelino and Josefina Concepcion have appealed to this court
ISSUE: W/N Aquilina Molales common-law wife and her illegitimate
children can claim the benefits
HELD: YES.
 Juanita Golpeo, by her silence and actions, had acquiesced in the
illicit relations between her husband and appellee Aquilina Maloles
 new Civil Code recognized certain successional rights of illegitimate
children
Separate Opinions:
 REYES, J.B.L., J., concurring
 I concur in the result for the reason that the contract here involved
was perfected before the new Civil Code took effect, and hence its
provisions cannot be made to apply retroactively

SSS v. Davac - SSS Benefits

Facts:

> Davac was an SSS member, and designated Candelaria Davac, his
alleged wife, as his beneficiary.
> When he died, both his first wife, Lourdes and his second wife,
Candelaria filed claims for the death benefits.

> Due to the conflicting claims, the SSS filed a petition praying that
both of them be required to interplead and litigate the conflicting
claims.

> The death benefits were awarded to Candelaria Davac.

Issue:

Who is entitled to the SSS benefits?

Held:

Candelaria.

Under the SSS Act, the beneficiary as recorded by the employee’s


employer is the one entitled to the death benefits, hence they should
go to Candelaria. Lourdes contends that the designation made in the
person of Candelaria who is party in a bigamous marriage is null and
void for being against Art. 739 of the CC. SC held that the
disqualification mentioned in Art. 739 is NOT applicable to Candelaria,
because she was not guilty of concubinage , there bieing NO proof that
she had actual knowledge of the previous marriage of her husband.

Social Security System V. Davac (1966)

Lessons Applicable: Invalid Designation (Insurance)


FACTS:
 Petronilo Davac, a former employee of Lianga Bay Logging Co., Inc.
became a member of the Social Security System (SSS) he
designated Candelaria Davac as his beneficiary and indicated his
relationship to her as that of "wife"
 Lourdes Tuplano his legal wife and their son Romeo
Davac and Candelaria Davac and their minor daughter Elizabeth
Davac filed their claims
 Social Security Commission: Candelaria Davac
ISSUE: W/N Candelaria Davac can claim and New Civil Code 739 is not
applicable
HELD: YES.
 she was not guilty of concubinage, there being no proof that she
had knowledge of the previous marriage of her husband Petronilo
 The amounts that may thus be received cannot be considered as
property earned by the member during his lifetime
 if there is a named beneficiary and the designation is not invalid
(as it is not so in this case), it is not the heirs of the employee who
are entitled to receive the benefits (unless they are the designated
beneficiaries themselves). It is only when there is no designated
beneficiaries or when the designation is void, that the laws of
succession are applicable. And we have already held that the Social
Security Act is not a law of succession.

Vda. De Consuegra v. GSIS - Retirement Insurance Benefits

Facts:

> Jose Consuegra was employed as a shop foreman of the Office of


the District Engineer in Surigao Del Norte.

> When he was still alive, he contracted two marriages:

o First – Rosario Diaz; 2 children = Jose Consuegra Jr. and Pedro but
both predeceased him

o 2nd – Basilia Berdin; 7 children. (this was contracted in GF while


the first marriage subsisted)

> Being a GSIS member when he died, the proceeds of his life
insurance were paid by the GSIS to Berdin and her children who were
the beneficiaries named in the policy.

> Since he was in the gov’t service for 22.5028 years, he was entitled
to retirement insurance benefits, for which no beneficiary was
designated.
> Both families filed their claims with the GSIS, which ruled that the
legal heirs were Diaz who is entitled to one-half or 8/16 of the
retirement benefits and Berdin and her children were entitled to the
remaining half, each to receive an equal share of 1/16.

> Berdin went to CFI on appeal. CFI affirmed GSIS decision.

Issue:

To whom should the retirement insurance benefits be paid?

Held:

Both families are entitled to half of the retirement benefits.

The beneficiary named in the life insurance does NOT automatically


become the beneficiary in the retirement insurance. When Consuegra,
during the early part of 1943, or before 1943, designated his
beneficiaries in his life insurance, he could NOT have intended those
beneficiaries of his life insurance as also the beneficiaries of his
retirement insurance because the provisions on retirement insurance
under the GSIS came about only when CA 186 was amended by RA
660 on June 18, 1951.

Sec. 11(b) clearly indicates that there is need for the employee to file
an application for retirement insurance benefits when he becomes a
GSIS member and to state his beneficiary. The life insurance and the
retirement insurance are two separate and distinct systems of benefits
paid out from 2 separate and distinct funds.

In case of failure to name a beneficiary in an insurance policy, the


proceeds will accrue to the estate of the insured. And when there
exists two marriages, each family will be entitled to one-half of the
estate.

Vda. Dde Consuegra V. Governments Service Insurance System


(1971)

Lessons Applicable: Invalid Designation (Insurance)

FACTS:
 Jose Consuegra contracted two marriages.
 First with Rosario Diaz
 2 children who predeceased their father: Jose Consuegra, Jr. and
Pedro Consuegra
 Second with Basilia Berdin while marriage is still subsisting
 7 children: Juliana, Pacita, Maria Lourdes, Jose, Rodrigo, Lenida
and Luz, all surnamed Consuegra
 Consuegra did not designate any beneficiary who would receive the
retirement insurance benefits due to him
 Rosario Diaz filed a claim with the GSIS asking that the retirement
insurance benefits be paid to her as the only legal heir of
Consuegra, considering that the deceased did not designate any
beneficiary
 GSIS: legal heirs were Rosario Diaz (1/2 or 8/16), Basilia Berdin
and their seven children (1/2 or 8/16) (1/16 each)
 Basilia Berdin and her children filed petition for mandamus with
preliminary injunction in CFI
 RTC: dismissed the case
 Basilia Berdin and her children appealed contending that because
the deceased Jose Consuegra failed to designate the beneficiaries
in his retirement insurance, the appellants who were the
beneficiaries named in the life insurance should automatically be
considered the beneficiaries to receive the retirement insurance
benefits (Thus, excluding Rosario Diaz)
ISSUE: W/N the beneficiaries named in the life insurance should
automatically be considered the beneficiaries to receive the retirement
insurance benefits

HELD: NO. appealed from is affirmed


 The insured in a life insurance may designate any person as
beneficiary unless disqualified to be so under the provisions of the
Civil Code
 And in the absence of any beneficiary named in the life insurance
policy, the proceeds of the insurance will go to the estate of the
insured
 Retirement insurance is primarily intended for the benefit of the
employee — to provide for his old age, or incapacity, after
rendering service in the government for a required number of
years. If the employee reaches the age of retirement, he gets the
retirement benefits even to the exclusion of the beneficiary or
beneficiaries named in his application for retirement insurance. The
beneficiary of the retirement insurance can only claim the proceeds
of the retirement insurance if the employee dies before
retirement. If the employee failed or overlooked to state the
beneficiary of his retirement insurance, the retirement benefits will
accrue to his estate and will be given to his legal heirs in
accordance with law, as in the case of a life insurance if no
beneficiary is named in the insurance policy.
 with respect to the right of the second wife, although the second
marriage can be presumed to be void ab initio as it was celebrated
while the first marriage was still subsisting, still there is need for
judicial declaration of such nullity

INSURABLE INTEREST

Col. C. Castro v. Insurance Commissioner - Insurable Interest

Facts:

> Castro applied for insurance on the life of his driver. On the basis
of such application, Insular Life issued policy No. 934943 effective July
18, 1979.

> The policy applied for and issued was on a 20-yr endowment plan
for the sum of P25T with double indemnity in case of accidental death.
> Castro paid the first quarterly premium of P309.95. About 3
months later, on Oct. 16, 1959, the insured driver was allegedly shot
to death by unknown persons. (hmmm… sounds fishy…)

> Castro then filed a claim for the total benefits of 50T under the
policy.

> Insular life denied the claim on the ground that the policy was
VOID. Insular instead refunded to Castro the premiums he had paid.

Issue:Whether or not Castro has an insurable interest in his driver.

Held:NO.

The requirement of insurable interest to support a contract of


insurance is based upon consideration of public policy which renders
wager policies INVALID. To sustain a contract of this character it must
appear that there is a real concern in the life of the party whose death
would be the cause of substantial loss to those who are named as a
beneficiary.

Mere relationship of uncle and nephew, employer and employee is NOT


sufficient to provide an insurable interest on the life of the insured. It
must be shown that the destruction of the life of the insured would
cause pecuniary loss to the complainant. This, Castro failed to prove.

Consuegra v GSIS G.R. No. L-28093 January 30, 1971

Facts:
Appeal on purely questions of law from the decision of the Court of
First Instance of Surigao del Norte, dated March 7, 1967, in its Special
Proceeding No. 1720.
The late Jose Consuegra was employed as a shop foreman in the
province of Surigao del Norte. He contracted two marriages, the first
with Rosario Diaz and the second, which was contracted in good faith
while the first marriage was subsisting, with Basilia Berdin.
Consuegra died, while the proceeds of his GSIS life insurance were
paid to petitioner Basilia Berdin and her children who were the
beneficiaries named in the policy. They received Php 6,000.
Consuegra did not designate any beneficiary who would receive the
retirement insurance benefits due to him. Respondent Rosario Diaz,
the widow by the first marriage, filed a claim with the GSIS asking that
the retirement insurance benefits be paid to her as the only legal heir
of Consuegra, considering that the deceased did not designate any
beneficiary with respect to his retirement insurance benefits.
Petitioner Berdin and her children, likewise, filed a similar claim with
the GSIS, asserting that being the beneficiaries named in the life
insurance policy of Consuegra, they are the only ones entitled to
receive the retirement insurance benefits due the deceased
Consuegra.
The GSIS ruled that the legal heirs of the late Jose Consuegra were
Rosario Diaz, his widow by his first marriage who is entitled to one-
half, or 8/16, of the retirement insurance benefits, on the one hand;
and Basilia Berdin, his widow by the second marriage and their seven
children, on the other hand, who are entitled to the remaining one-
half, or 8/16.
Basilia Berdin didn’t agree. She filed a petition declaring her and her
children to be the legal heirs and exclusive beneficiaries of the
retirement insurance.
The trial court affirmed stating that: "when two women innocently and
in good faith are legally united in holy matrimony to the same man,
they and their children, born of said wedlock, will be regarded as
legitimate children and each family be entitled to one half of the
estate.”
Hence the present appeal by Basilia Berdin and her children.

Issue: To whom should this retirement insurance benefits of Jose


Consuegra be paid, because he did not designate the beneficiary of his
retirement insurance?

Held: No. Petition denied.

Ratio:
Berdin averred that because the deceased Jose Consuegra failed to
designate the beneficiaries in his retirement insurance, the appellants
who were the beneficiaries named in the life insurance should
automatically be considered the beneficiaries to receive the retirement
insurance benefits.
The GSIS offers two separate and distinct systems of benefits to its
members — one is the life insurance and the other is the retirement
insurance. These two distinct systems of benefits are paid out from
two distinct and separate funds that are maintained by the GSIS.
In the case of the proceeds of a life insurance, the same are paid to
whoever is named the beneficiary in the life insurance policy. As in the
case of a life insurance provided for in the Insurance Act, the
beneficiary in a life insurance under the GSIS may not necessarily be a
heir of the insured. The insured in a life insurance may designate any
person as beneficiary unless disqualified to be so under the provisions
of the Civil Code. And in the absence of any beneficiary named in the
life insurance policy, the proceeds of the insurance will go to the estate
of the insured.
Retirement insurance is primarily intended for the benefit of the
employee, to provide for his old age, or incapacity, after rendering
service in the government for a required number of years. If the
employee reaches the age of retirement, he gets the retirement
benefits even to the exclusion of the beneficiary or beneficiaries
named in his application for retirement insurance. The beneficiary of
the retirement insurance can only claim the proceeds of the retirement
insurance if the employee dies before retirement. If the employee
failed or overlooked to state the beneficiary of his retirement
insurance, the retirement benefits will accrue to his estate and will be
given to his legal heirs in accordance with law, as in the case of a life
insurance if no beneficiary is named in the insurance policy.
GSIS had correctly acted when it ruled that the proceeds should be
divided equally between his first living wife and his second. The lower
court has correctly applied the ruling of this Court in the case of Lao v
Dee.
Gomez vs. Lipana- in construing the rights of two women who were
married to the same man, held "that since the defendant's first
marriage has not been dissolved or declared void the conjugal
partnership established by that marriage has not ceased. Nor has the
first wife lost or relinquished her status as putative heir of her husband
under the new Civil Code, entitled to share in his estate upon his death
should she survive him. Consequently, whether as conjugal partner in
a still subsisting marriage or as such putative heir she has an interest
in the husband's share in the property here in dispute....
With respect to the right of the second wife, although the second
marriage can be presumed to be void ab initio as it was celebrated
while the first marriage was still subsisting, still there is need for
judicial declaration of such nullity. And inasmuch as the conjugal
partnership formed by the second marriage was dissolved before
judicial declaration of its nullity, "the only lust and equitable solution in
this case would be to recognize the right of the second wife to her
share of one-half in the property acquired by her and her husband and
consider the other half as pertaining to the conjugal partnership of the
first marriage."

Lincoln National Life v. San Juan - Life Insurance

Facts:

> An employer insured the life of the employee with two insurance
companies.

> The insurance totaled 200T and the only beneficiaries were the
employer and his wife.

> A severed head was later found, purportedly that of the insured
employee.

> The insurance companies refused to pay on the ground that the
employer had no insurable interest in the life of the employee.

Issue:

Whether or not the employer can recover the proceeds of a life


insurance policy of his employee.

Held:NO.

The insured was a tenant in a coconut land owned by the employer


and his earning were barely that of a farm laborer. It was established
that the insured could not have afforded the insurance policies drawn
on his life. Many more policies were found to have been issued with
the employee/tenant as insured and the employer and his wife as
beneficiaries.

The policies were also found to have been acquired in quick


succession. It was found that the various postal money orders issued
in payment of the premiums were made by the employer. It appears
that, based on the circumstances and evidence, the insurance was
really taken out by the employer.
Gercio v. Sun Life - Insurance Beneficiary

Facts:

> Sunlife issued a life insurance policy to Gercio, the former agreeing
to insure the life of Gercio for 2T to be paid to him on Feb. 1, 1930 or
if he should die before said date, then to his wife Andrea, should she
survive him; otherwise to the executor, administrator of Gercio.

> The policy did not include any provision reserving to Gercio the
right to change the beneficiary.

> The wife was convicted of adultery and a decree of divorce was
issued.

> Gercio notified Sunlife that he had revoked his donation in favor of
Andrea and that he had designated his present wife Adela as his
beneficiary.

> Sunlife refused to change the beneficiary.

Issue:

Whether or not Gercio may change the beneficiary in the policy.

Held:NO.

If the policy contains no provision authorizing a change of beneficiary


without the beneficiary’s consent, the insured cannot make such
change. It is held that a life insurance policy of a husband made
payable to his wife as a beneficiary is the separate property of the
beneficiary and beyond the control of the husband. (NOTE: this case is
based on the old rule under the Insurance Act)

Court also held that the designation of a beneficiary that is originally


valid does NOT render it invalid but to a subsequent cessation of the
interests between the beneficiary and insured.

Gercio V. Sun Life Assurance Co. Of Canada (1925)

Lessons Applicable:
 Blood relationship (Insurance)
 Revocable Designation (Insurance)
FACTS:
 January 29, 1910: Sun Life Assurance Co. of Canada issued a 20-
year endowment insurance policy on the life of Hilario Gercio
 insurance company agreed to insure the life of Gercio for the sum
of P2,000, to be paid him on February 1, 1930, or if the insured
should die before said date, then to his wife, Mrs. Andrea Zialcita,
should she survive him; otherwise to the executors, administrators,
or assigns of the insured
 policy did not include any provision reserving to the insured the
right to change the beneficiary
 End of 1919: she was convicted of the crime of adultery
 September 4, 1920: a decree of divorce was issued
 March 4, 1922: Gercio formally notified the Sun Life that he had
revoked his donation in favor of Andrea Zialcita, and that he had
designated in her stead his present wife, Adela Garcia de Gercio, as
the beneficiary of the policy
 Sun Life refused
 Gercio filed a petition for mandamus to compel Sun Life
 Trial Court: favored Gercio
ISSUE: W/N Gercio has the right to change the beneficiary of the
policy
HELD: NO. Dismissed.
 The wife has an insurable interest in the life of her husband.
 The beneficiary has an absolute vested interest in the policy from
the date of its issuance and delivery. So when a policy of life
insurance is taken out by the husband in which the wife is named
as beneficiary, she has a subsisting interest in the policy
 applies to a policy to which there are attached the incidents of a
loan value, cash surrender value, an automatic extension by
premiums paid, and to an endowment policy, as well as to an
ordinary life insurance policy.
 If the husband wishes to retain to himself the control and
ownership of the policy he may so provide in the policy.
 But if the policy contains no provision authorizing a change of
beneficiary without the beneficiary's consent, the insured cannot
make such change.
 Accordingly, it is held that a life insurance policy of a husband
made payable to the wife as beneficiary, is the separate property of
the beneficiary and beyond the control of the husband.
 effect produced by the divorce, the Philippine Divorce Law, Act No.
2710, merely provides in section 9 that the decree of divorce shall
dissolve the community property as soon as such decree becomes
final
 absence of a statute to the contrary, that if a policy is taken out
upon a husband's life the wife is named as beneficiary therein, a
subsequent divorce does not destroy her rights under the policy
 Neither the husband, nor the wife, nor both together had power to
destroy the vested interest of the children in the policy.
Separate Opinion:
 Johnson, Concurring Opinion:
 I agree with the majority of the court, that the judgment of the
lower court should be revoked, but for a different reason. In my
judgment, the action is premature and should have been
dismissed.

El Oriente v. Posadas - Taxability of Insurance Proceeds

Facts:

> El Oriente in order to protect itself against the loss that it might
suffer by reason of the death of its manager, A. Velhagen, who had
had more than thirty-five (35) years of experience in the manufacture
of cigars in the Philippines, procured from the Manufacturers Life
Insurance Co., of Toronto, Canada, thru its local agent E. E. Elser, an
insurance policy on the life of the said A. Velhagen for the sum of
$50,000, United States currency designating itself as the beneficiary.

> El Oriente paid for the premiums due thereon and charged as
expenses of its business all the said premiums and deducted the same
from its gross incomes as reported in its annual income tax returns,
which deductions were allowed upon a showing that such premiums
were legitimate expenses of its business.

> Upon the death of A. Velhagen in 1929, the El Oriente received all
the proceeds of the said life insurance policy, together with the
interests and the dividends accruing thereon, aggregating P104,957.88

> CIR assessed El Oriente for deficiency taxes because El Oriente did
not include as income the proceeds received from the insurance.

Issue:

Whether or not the proceeds of insurance taken by a corporation on


the life of an important official to indemnify it against loss in case of
his death, are taxable as income under the Philippine Income Tax Law

Held:NOT TAXABLE.

In Chapter I of the Tax Code, is to be found section 4 which provides


that, "The following incomes shall be exempt from the provisions of
this law: (a) The proceeds of life insurance policies paid to
beneficiaries upon the death of the insured . . ." Section 10, as
amended, in Chapter II On Corporations, provides that, "There shall be
levied, assessed, collected, and paid annually upon the total net
income received in the preceding calendar year from all sources by
every corporation . . .a tax of three per centum upon such income .
. ." Section 11 in the same chapter, provides the exemptions under the
law, but neither here nor in any other section is reference made to the
provisions of section 4 in Chapter I.

Under the view we take of the case, it is sufficient for our purposes to
direct attention to the anomalous and vague condition of the law. It is
certain that the proceeds of life insurance policies paid to individual
beneficiaries upon the death of the insured are exempt. It is not so
certain that the proceeds of life insurance policies paid to corporate
beneficiaries upon the death of the insured are likewise exempt. But at
least, it may be said that the law is indefinite in phraseology and does
not permit us unequivocally to hold that the proceeds of life insurance
policies received by corporations constitute income which is taxable

It will be recalled that El Oriente, took out the insurance on the life of
its manager, who had had more than thirty-five years' experience in
the manufacture of cigars in the Philippines, to protect itself against
the loss it might suffer by reason of the death of its manager. We do
not believe that this fact signifies that when the plaintiff received
P104,957.88 from the insurance on the life of its manager, it thereby
realized a net profit in this amount. It is true that the Income Tax Law,
in exempting individual beneficiaries, speaks of the proceeds of life
insurance policies as income, but this is a very slight indication of
legislative intention. In reality, what the plaintiff received was in the
nature of an indemnity for the loss which it actually suffered because
of the death of its manager.

El Oriente, Fabrica De Tabacos, Inc., V. Posadas (1931)

Lessons Applicable: Pecuniary Interest (Insurance)


FACTS:
 March 18, 1925: El Oriente, Fabrica de Tabacos, Inc. in order to
protect itself against the loss that it might suffer by reason of the
death of its manager, A. Velhagen, who had more than 35 years of
experience in the manufacture of cigars in the Philippine Islands,
and whose death would be a serious loss procured from the
Manufacturers Life Insurance Co., of Toronto, Canada, thru its local
agent E.E. Elser, an insurance policy on the life of A. Velhagen for
$50,000
 designated itself as the sole beneficiary
 Upon the death of A. Velhagen in the year 1929, El
Oriente received all the proceeds of the life insurance policy,
together with the interests and the dividends accruing thereon,
aggregating P104,957.88
 Collector of Internal Revenue assessed and levied the sum of
P3,148.74 as income tax on the proceeds of the insurance
policy which tax El Oriente paid
ISSUE: W/N proceeds of life insurance policies paid to corporate
beneficiaries upon the death of the insured are also exempted

HELD: YES. reversed and favoring El Oriente


 In reality, what the plaintiff received was in the nature of an
indemnity for the loss which it actually suffered because of the
death of its manager and not taxable income

Philamcare v. CA- Health Care Agreement

Facts:

> Ernani Trinos, applied for a health care coverage with Philamcare.
In the standard application form, he answered NO to the following
question: “Have you or any of your family members ever consulted or
been treated for high blood pressure, heart trouble, diabetes, cancer,
liver disease, asthma or peptic ulcer? (If Yes, give details)”

> The application was approved for a period of one year from March
1, 1988 to March 1, 1989. He was a issued Health Care Agreement,
and under such, he was entitled to avail of hospitalization benefits,
whether ordinary or emergency, listed therein. He was also entitled to
avail of "out-patient benefits" such as annual physical examinations,
preventive health care and other out-patient services.

> Upon the termination of the agreement, the same was extended for
another year from March 1, 1989 to March 1, 1990, then from March
1, 1990 to June 1, 1990. The amount of coverage was increased to a
maximum sum of P75,000.00 per disability.
> During the period of his coverage, Ernani suffered a heart attack
and was confined at the Manila Medical Center (MMC) for one month
beginning March 9, 1990.

> While her husband was in the hospital, Julita tried to claim the
benefits under the health care agreement. However, Philamcare
denied her claim saying that the Health Care Agreement was void.

> According to Philamcare, there was concealment regarding Ernani's


medical history.

 Doctors at the MMC allegedly discovered at the time of Ernani's


confinement that he was hypertensive, diabetic and asthmatic,
contrary to his answer in the application form.

> Julita had no choice but to pay the hospitalization expenses herself,
amounting to about P76,000.00

> After her husband was discharged from the MMC, he was attended
by a physical therapist at home. Later, he was admitted at the Chinese
General Hospital (CGH). Due to financial difficulties, Julita brought her
husband home again. In the morning of April 13, 1990, Ernani had
fever and was feeling very weak. Julita was constrained to bring him
back to the CGH where he died on the same day.

> Julita instituted, an action for damages against Philamcare. She


asked for reimbursement of her expenses plus moral damages and
attorney's fees. RTC decided in favor of Julita. CA affirmed.

Issues and Resolutions:

Philamcare brought the instant petition for review, raising the primary
argument that a health care agreement is not an insurance contract;
hence the "incontestability clause" under the Insurance Code Title 6,
Sec. 48 does not apply.

SC held that in the case at bar, the insurable interest of respondent's


husband in obtaining the health care agreement was his own health.
The health care agreement was in the nature of non-life insurance,
which is primarily a contract of indemnity. Once the member incurs
hospital, medical or any other expense arising from sickness, injury or
other stipulated contingent, the health care provider must pay for the
same to the extent agreed upon under the contract.

Under the title Claim procedures of expenses, Philamcare. had 12 mos


from the date of issuance of the Agreement within which to contest the
membership of the patient if he had previous ailment of asthma, and
six months from the issuance of the agreement if the patient was sick
of diabetes or hypertension. The periods having expired, the defense
of concealment or misrepresentation no longer lie.

Petitioner argues that respondent's husband concealed a material fact


in his application. It appears that in the application for health
coverage, petitioners required respondent's husband to sign an
express authorization for any person, organization or entity that has
any record or knowledge of his health to furnish any and all
information relative to any hospitalization, consultation, treatment or
any other medical advice or examination.

Philamcare cannot rely on the stipulation regarding "Invalidation of


agreement" which reads:

Failure to disclose or misrepresentation of any material information by


the member in the application or medical examination, whether
intentional or unintentional, shall automatically invalidate the
Agreement from the very beginning and liability of Philamcare shall be
limited to return of all Membership Fees paid. An undisclosed or
misrepresented information is deemed material if its revelation would
have resulted in the declination of the applicant by Philamcare or the
assessment of a higher Membership Fee for the benefit or benefits
applied for.

The answer assailed by petitioner was in response to the question


relating to the medical history of the applicant. This largely depends on
opinion rather than fact, especially coming from respondent's husband
who was not a medical doctor. Where matters of opinion or judgment
are called for, answers made in good faith and without intent to
deceive will not avoid a policy even though they are untrue. Thus,

(A)lthough false, a representation of the expectation, intention, belief,


opinion, or judgment of the insured will not avoid the policy if there is
no actual fraud in inducing the acceptance of the risk, or its
acceptance at a lower rate of premium, and this is likewise the rule
although the statement is material to the risk, if the statement is
obviously of the foregoing character, since in such case the insurer is
not justified in relying upon such statement, but is obligated to make
further inquiry. There is a clear distinction between such a case and
one in which the insured is fraudulently and intentionally states to be
true, as a matter of expectation or belief, that which he then knows, to
be actually untrue, or the impossibility of which is shown by the facts
within his knowledge, since in such case the intent to deceive the
insurer is obvious and amounts to actual fraud.

The fraudulent intent on the part of the insured must be established to


warrant rescission of the insurance contract. Concealment as a defense
for the health care provider or insurer to avoid liability is an affirmative
defense and the duty to establish such defense by satisfactory and
convincing evidence rests upon the provider or insurer. In any case,
with or without the authority to investigate, petitioner is liable for
claims made under the contract. Having assumed a responsibility
under the agreement, petitioner is bound to answer the same to the
extent agreed upon. In the end, the liability of the health care provider
attaches once the member is hospitalized for the disease or injury
covered by the agreement or whenever he avails of the covered
benefits which he has prepaid.

Under Section 27 of the Insurance Code, "a concealment entitles the


injured party to rescind a contract of insurance." The right to rescind
should be exercised previous to the commencement of an action on
the contract. In this case, no rescission was made. Besides, the
cancellation of health care agreements as in insurance policies require
the concurrence of the following conditions:

1. Prior notice of cancellation to insured;

2. Notice must be based on the occurrence after effective date of


the policy of one or more of the grounds mentioned;

3. Must be in writing, mailed or delivered to the insured at the


address shown in the policy;
4. Must state the grounds relied upon provided in Section 64 of the
Insurance Code and upon request of insured, to furnish facts on
which cancellation is based.

None of the above pre-conditions was fulfilled in this case. When the
terms of insurance contract contain limitations on liability, courts
should construe them in such a way as to preclude the insurer from
non-compliance with his obligation. Being a contract of adhesion, the
terms of an insurance contract are to be construed strictly against the
party which prepared the contract — the insurer. By reason of the
exclusive control of the insurance company over the terms and
phraseology of the insurance contract, ambiguity must be strictly
interpreted against the insurer and liberally in favor of the insured,
especially to avoid forfeiture. This is equally applicable to Health Care
Agreements. The phraseology used in medical or hospital service
contracts, such as the one at bar, must be liberally construed in favor
of the subscriber, and if doubtful or reasonably susceptible of two
interpretations the construction conferring coverage is to be adopted,
and exclusionary clauses of doubtful import should be strictly
construed against the provider.

Philamcare Health Systems, Inc. V. CA (2002)

Lessons Applicable:
 Elements (Insurance)
 Blood Relationship (Insurance)
FACTS:
 Ernani Trinos, deceased husband of Julita Trinos, applied for a
health care coverage with Philamcare Health Systems, Inc.
 He answered the standard application form: Have you or any of
your family members ever consulted or been treated for high blood
pressure, heart trouble, diabetes, cancer, liver disease, asthma or
peptic ulcer? (If Yes, give details). - NO
 the application was approved for a period of one year from March
1, 1988 to March 1, 1989. Accordingly, he was issued Health Care
Agreement No. P010194
 Under the agreement, respondent’s husband was entitled to avail
of hospitalization benefits, whether ordinary or emergency, listed
therein. He was also entitled to avail of "out-patient benefits" such
as annual physical examinations, preventive health care and other
out-patient services.
 Upon the termination of the agreement, the same was extended for
another year from March 1, 1989 to March 1, 1990, then from
March 1, 1990 to June 1, 1990. The amount of coverage was
increased to a maximum sum of P75,000.00 per disability.
 During the period of his coverage, Ernani suffered a heart attack
and was confined at the Manila Medical Center (MMC) for 1 month
beginning March 9, 1990.
 While her husband was in the hospital, Julina Trinos tried to claim
the benefits under the health care agreement.
 Philamcare denied her claim saying that the Health Care Agreement
was void for concealing Ernani’s medical history so she paid the
hospitalization expenses of P76,000.00 herself.
 Doctors at the MMC allegedly discovered at the time of Ernani’s
confinement that he was hypertensive, diabetic and asthmatic,
contrary to his answer in the application form.
 After being discharged from the MMC, he was attended by a
physical therapist at home.
 Later, he was admitted at the Chinese General Hospital.
 Due to financial difficulties, however, he was brought home again.
 April 13, 1990 morning: Ernani had fever and was feeling very
weak
 He was brought to Chinese General Hospital where he died
 July 24, 1990: She brought action for damages against Philamcare
Health Systems Inc. and its president, Dr. Benito Reverente
 RTC: Philamcare and Dr. Benito Reverent to pay and
reimburse P76k plus interest, moral damages, exemplary damages,
attorney's fees and cost of suit
 CA: affirmed the decision of RTC but deleted all awards for
damages and absolved Philamcare
 Philamcare brought an instant petition for review arguing that:
 health care agreement is not an insurance contract; hence the
"incontestability clause" under the Insurance Code does not apply.
 grants "living benefits," such as medical check-ups and
hospitalization which a member may immediately enjoy so long as
he is alive upon effectivity of the agreement until its expiration
one-year thereafter
 only medical and hospitalization benefits are given under the
agreement without any indemnification, unlike in an insurance
contract where the insured is indemnified for his loss
 since Health Care Agreements are only for a period of one year, as
compared to insurance contracts which last longer; incontestability
clause does not apply, as the same requires an effectivity period of
at least two years
 insurance company is governed by the Insurance Commission, but
a Health Maintenance Organization under the authority of the
Department of Health
ISSUE:

1. W/N the health care agreement is a contract of insurance. - YES


2. W/N the spouse being "not" legal wife can claim - YES

HELD: Petition is DENIED. CA AFFIRMED.


1. YES.

P.D. 612 Insurance Code


Sec. 2 (1)
(1) A "contract of insurance" is an agreement whereby one
undertakes for a consideration to indemnify another against loss,
damage or liability arising from an unknown or contingent event.
Sec. 3
Sec. 3. Any contingent or unknown event, whether past or future,
which may damnify a person having an insurable interest, or create
a liability against him, may be insured against, subject to the
provisions of this chapter.

The consent of the husband is not necessary for the validity of an


insurance policy taken out by a married woman on her life or that of
her children.

Any minor of the age of eighteen years or more, may,


notwithstanding such minority, contract for life, health and accident
insurance, with any insurance company duly authorized to do
business in the Philippines, provided the insurance is taken on his
own life and the beneficiary appointed is the minor's estate or the
minor's father, mother, husband, wife, child, brother or sister.

The married woman or the minor herein allowed to take out an


insurance policy may exercise all the rights and privileges of an
owner under a policy.

All rights, title and interest in the policy of insurance taken out by an
original owner on the life or health of a minor shall automatically
vest in the minor upon the death of the original owner, unless
otherwise provided for in the policy.

 In the case at bar, the insurable interest of respondent's husband


in obtaining the health care agreement was his own health.
 in the nature of non-life insurance, which is primarily a contract of
indemnity
 Once the member incurs hospital, medical or any other expense
arising from sickness, injury or other stipulated contingent, the
health care provider must pay for the same to the extent agreed
upon under the contract.
 The answer in response to the question relating to the medical
history of the applicant largely depends on opinion rather than fact,
especially coming from respondent's husband who was not a
medical doctor.
 Where matters of opinion or judgment are called for, answers made
in good faith and without intent to deceive will not avoid a policy
even though they are untrue.
 The fraudulent intent on the part of the insured must be
established to warrant rescission of the insurance contract.
 Concealment as a defense for the health care provider or insurer to
avoid liability is an affirmative defense and the duty to establish
such defense by satisfactory and convincing evidence rests upon
the provider or insurer.
P.D. 612 Insurance Code
Sec. 27
Sec. 27. A concealment whether intentional or unintentional entitles
the injured party to rescind a contract of insurance.
 cancellation of health care agreements as in insurance policies
require the concurrence of the following conditions: - none of these
was made
1. Prior notice of cancellation to insured;
2. Notice must be based on the occurrence after effective date of
the policy of one or more of the grounds mentioned;
3. Must be in writing, mailed or delivered to the insured at the
address shown in the policy;
4. Must state the grounds relied upon provided in Section 64 of the
Insurance Code and upon request of insured, to furnish facts on
which cancellation is based.
 When the terms of insurance contract contain limitations on
liability, courts should construe them in such a way as to preclude
the insurer from non-compliance with his obligation.
 Being a contract of adhesion, the terms of an insurance contract
are to be construed strictly against the party which prepared the
contract - the insurer.
 (U)nder the title Claim procedures of expenses, the defendant
Philamcare Health Systems Inc. had twelve months from the date
of issuance of the Agreement within which to contest the
membership of the patient if he had previous ailment of asthma,
and six months from the issuance of the agreement if the patient
was sick of diabetes or hypertension. The periods having expired,
the defense of concealment or misrepresentation no longer lie.
2. YES.

P.D. 612 Insurance Code


Sec. 10
Sec. 10. Every person has an insurable interest in the life and
health:
(1) of himself, of his spouse and of his children;
(2) of any person on whom he depends wholly or in part for
education or support, or in whom he has a pecuniary interest;
(3) of any person under a legal obligation to him for the payment of
money, respecting property or service, of which death or illness
might delay or prevent the performance; and
(4) of any person upon whose life any estate or interest vested in
him depends.

 not the legal wife (deceased was previously married to another


woman who was still alive)
 health care agreement is in the nature of a contract of indemnity.
 payment should be made to the party who incurred the expenses

INSURABLE INTEREST IN PROPERTY

Harvardian Colleges v. Country Bankers Insurance Corp.

Facts:

> Harvardian is a family corporation, the stockholders of which are


Ildefonso Yap, Virginia King Yap and their children.

> Prior to Aug. 9, 1979, an agent of Country Bankers proposed to


Harvardian to insure its school building. Although at first reluctant,
Harvardian agreed.
> Country Banks sent an inspector to inspect the school building and
agreed to insure the same for P500,000 for which Harvardian paid an
annual premium of P2,500.

> On Aug. 9, 1979, Country Bankers issued to Harvardian a fire


insurance policy. On March 12, 1980, (39 days before I was born…
hehehehe )during the effectivity of said insurance policy, the insured
property was totally burned rendering it a total loss.

> A claim was made by plaintiff upon defendant but defendant denied
it contending that plaintiff had no insurable interest over the building
constructed on the piece of land in the name of the late Ildefonso Yap
as owner.

> It was contended that both the lot and the building were owned by
Ildefonso Yap and NOT by the Harvardian Colleges.

Issue:

Whether or not Harvardian colleges has a right to the proceeds.

Held:

Harvardian has a right to the proceeds.

Regardless of the nature of the title of the insured or even if he did not
have title to the property insured, the contract of fire insurance should
still be upheld if his interest in or his relation to the property is such
that he will be benefited in its continued existence or suffer a direct
pecuniary loss from its destruction or injury. The test in determining
insurable interest in property is whether one will derive pecuniary
benefit or advantage from its preservation, or will suffer pecuniary loss
or damage from its destruction, termination or injury by the happening
of the event insured against.

Here Harvardian was not only in possession of the building but was in
fact using the same for several years with the knowledge and consent
of Ildefonso Yap. It is reasonably fair to assume that had the building
not been burned, Harvardian would have been allowed the continued
use of the same as the site of its operation as an educational
institution. Harvardian therefore would have been directly benefited
by the preservation of the property, and certainly suffered a pecuniary
loss by its being burned.

Traders Insurance and Surety Co. v. Golangco- Insurance Proceeds

Facts:

> A decision was rendred in Civil Case No. 6306 granting Golangco
the right to collect rentals from a building in Sta. Cruz, Manila.

> Golangco then sought fire insurance from Traders. Before the
policy was issued, Golangco made a full and clear exposal of his
interests in the premises, i.e. that he was not the owner.

> The fire policy that defendant issued covered only all of Golangco’s
interest in the premises and his right to collect the rentals.

> The building burned down in a fire and Golangco sought to collect
from Traders. Traders denied any liability on the ground that since
Golangco was not the owner of the premises then he had no insurable
interest in the same and consequently, he could not collect the
insurance proceeds.

Issue:

Whether or not plaintiff can claim the insurance proceeds.

Held.YES.

Both at the time of the issuance of the policy and at the time of the
fire, plaintiff Golangco was in legal possession of the premises,
collecting rentals from its occupant. It seems plain that if the
premises were destroyed as they were, by fire, Golangco would be, as
he was, directly damnified thereby; and hence he had an insurable
interest therein.

Traders Insurance & Surety Co. V. Golangco, Et Al (1954)

Lessons Applicable: Existing Interest (Insurance)


Laws Applicable: Sec. 13 of the Insurance Code

FACTS:
 Tomas Lianco and the Archbishop entered into a contract of lease
on a parcel of landowned by church
 As lessee, Lianco erected a building on the leased portion of the
church’s land.
 Lianco transferred ownership of this building to Kaw Eng
Si,who later transferred the same to Golangco.
 Transfers were made without the consent of the Archbishop
 The Archbishop filed an ejectment case against Lianco, who
appears to be occupants of the premises building with others
paying rent to Golangco.
 The right of Golangco to receive rent on the building was judicially
recognized in a case decided between Lianco and
others occupying the premises pursuant to a compromise
agreement.
 The Archbishop did not exercise his option to question Golangco’s
rights as lessee
 April 7,1949: Golangco applied for fire insurance with Trader’s
Insurance and Surety Co.
 fire insurance policy states: "that all insurancecovered under said
policy, includes the 'rent or
othersubject matter of insurance in respect of or inconnection with
any building or any property contained in any building"
 June 5, 1949: the building premises was burned so Golangco
requested Trader’s Insurance to pay the insurance amount of
10,000 including the amount of rent P1,100 monthly.
 Trader’s insurance refused to pay the insurance for the
rent averring that Golangco has no insurable interest
ISSUE: W/N Golangco has insurable interest
on the rent of the building premises which may lawfully/validly be
subject of insurance?

HELD: YES.
 Sec. 13 of the Insurance Code
Every interest in the property, whether real or personal, or any
relation thereto, or liability in respect thereof of such nature that a
contemplated peril might directly damnify the insured, is an insurable
interest.
 Both at the time of the issuance of the policy and at the time of the
fire, Golangco was in legal possession of the premises, collecting
rentals from its occupant.
 The argument of Trader’s Insurance that a policy of insurance must
specify the interest of the insured in the property insured, if he is
not the absolute owner thereof, is not meritorious because it was
the Trader’s, not Golangco, who prepared that policy, and it cannot
take advantage of its own acts to plaintiff's detriment; and, in any
case, this provisionwas substantially complied with by Golangco
when he made a full and clear statement of his interests to Trader's
manager.
 The contract between Lianco and the Archbishop only forbade
Lianco from transferring 'his rights as LESSEE but the contracts
Lianco made in favor of Kaw Eng Siand plaintiff Golangco did not
transfer such rights; hence no written consent thereto was
necessary. At worst, the contract would be voidable, but not a void
contract, at the option of the Archbishop and it does not appear
that it was ever exercised

Filipino Merchants v. CA- Insurable Interest

Facts:

> The Chao Tiek Seng a consignee of the shipment of fishmeal loaded
on board the vessel SS Bougainville and unloaded at the Port of Manila
on or about December 11, 1976 and seeks to recover from Filipino the
amount of P51,568.62 representing damages to said shipment which
has been insured by Filipino.
> Filipino brought a third party complaint against Compagnie Maritime
Des Chargeurs Reunis and/or E. Razon, Inc. seeking judgment against
the third party defendants in case judgment is rendered against it.

> It appears from the evidence presented that Chao insured said
shipment with Filipino for the sum of P267,653.59 for the goods
described as 600 metric tons of fishmeal in gunny bags of 90 kilos
each from Bangkok, Thailand to Manila against all risks under
warehouse to warehouse terms.

> Actually, what was imported was 59.940 metric tons not 600 tons
at $395.42 a ton.

> The fishmeal in 666 gunny bags were unloaded from the ship on
December 11, 1976 at Manila unto the arrastre contractor E. Razon,
Inc. and Filipino’s surveyor ascertained and certified that in such
discharge 105 bags were in bad order condition as jointly surveyed by
the ship's agent and the arrastre contractor.

> Based on said computation the Chao made a formal claim against
the Filipino for P51,568.62. A formal claim statement was also
presented by the plaintiff against the vessel, but the Filipino refused to
pay the claim.

Issues & Resolutions:

Filipino contends that an "all risks" marine policy has a technical


meaning in insurance in that before a claim can be compensable it is
essential that there must be "some fortuity," "casualty" or "accidental
cause" to which the alleged loss is attributable and the failure of herein
private respondent, upon whom lay the burden, to adduce evidence
showing that the alleged loss to the cargo in question was due to a
fortuitous event precludes his right to recover from the insurance
policy.

SC did not uphold this contention. An "all risks policy" should be read
literally as meaning all risks whatsoever and covering all losses by an
accidental cause of any kind. The terms "accident" and "accidental", as
used in insurance contracts, have not acquired any technical meaning.
They are construed by the courts in their ordinary and common
acceptance. Thus, the terms have been taken to mean that which
happens by chance or fortuitously, without intention and design, and
which is unexpected, unusual and unforeseen. An accident is an event
that takes place without one's foresight or expectation; an event that
proceeds from an unknown cause, or is an unusual effect of a known
cause and, therefore, not expected.

Coverage under an "all risks" provision of a marine insurance policy


creates a special type of insurance which extends coverage to risks not
usually contemplated and avoids putting upon the insured the burden
of establishing that the loss was due to the peril falling within the
policy's coverage; the insurer can avoid coverage upon demonstrating
that a specific provision expressly excludes the loss from coverage. A
marine insurance policy providing that the insurance was to be
"against all risks" must be construed as creating a special insurance
and extending to other risks than are usually contemplated, and
covers all losses except such as arise from the fraud of the
insured. The burden of the insured, therefore, is to prove merely that
the goods he transported have been lost, destroyed or deteriorated.
Thereafter, the burden is shifted to the insurer to prove that the loss
was due to excepted perils. To impose on the insured the burden of
proving the precise cause of the loss or damage would be inconsistent
with the broad protective purpose of "all risks" insurance.

In the present case, there being no showing that the loss was caused
by any of the excepted perils, the insurer is liable under the policy

Filipino contends that Chao does not have insurable interest, being
only a consignee of the goods.

Anent the issue of insurable interest, SC upheld the ruling of the CA


that Chao, as consignee of the goods in transit under an invoice
containing the terms under "C & F Manila," has insurable interest in
said goods.

Section 13 of the Insurance Code defines insurable interest in property


as every interest in property, whether real or personal, or any relation
thereto, or liability in respect thereof, of such nature that a
contemplated peril might directly damnify the insured. In principle,
anyone has an insurable interest in property who derives a benefit
from its existence or would suffer loss from its destruction whether he
has or has not any title in, or lien upon or possession of the
property. Insurable interest in property may consist in (a) an existing
interest; (b) an inchoate interest founded on an existing interest; or
(c) an expectancy, coupled with an existing interest in that out of
which the expectancy arises.

Chao, as vendee/consignee of the goods in transit has such existing


interest therein as may be the subject of a valid contract of insurance.
His interest over the goods is based on the perfected contract of
sale. The perfected contract of sale between him and the shipper of
the goods operates to vest in him an equitable title even before
delivery or before he performed the conditions of the sale. The
contract of shipment, whether under F.O.B., C.I.F., or C. & F. as in this
case, is immaterial in the determination of whether the vendee has an
insurable interest or not in the goods in transit. The perfected contract
of sale even without delivery vests in the vendee an equitable title, an
existing interest over the goods sufficient to be the subject of
insurance

Filipino Merchants Insurance Co. V. CA (1989)


Lessons Applicable: Existing Interest (Insurance)

Laws Applicable: Article 1523 of the Civil Code,Section 13 of the


Insurance Code
FACTS:

 Choa Tiek Seng, consignee of the shipment of fishmeal loaded,


insured in "all risks policy" 600 metric tons of fishmeal in new
gunny bags of 90 kilos each from Bangkok, Thailand to Manila
against all risks under warehouse to warehouse terms but
only 59.940 metric tons was imported
 When it was unloaded unto the arrastre contractor E. Razon, Inc.
and Filipino Merchants's surveyor ascertained and certified that in
such discharge 105 bags were in bad order condition which was
reflected in the survey report of Bad Order cargoes
 Before delivery to Choa, E. Razon's Bad Order Certificate showed
that a total of 227 bags in bad order condition
 Choa brought an action against Filipino Merchants Insurance Co.
who brought a third party complaint against Compagnie Maritime
Des Chargeurs Reunis and/or E. Razon, Inc.
 RTC: Ordered Filipino Merchants to pay Choa and
reimbursefrom Compagnie Maritime Des Chargeurs Reunis and
third party defendant E. Razon, Inc.
 CA: Affirmed but modified by adjudicating the third party complaint
 Filipino Merchants contended that Chao has no insurable interest
and therefore the policy should be void and that it was fraud that it
did not disclose of such fact
ISSUE: W/N Choa Tiek Seng as consignee of the shipment has
insurable interest

HELD: YES. CA affirmed.

 GR: the burden of proof is upon the insured to show that a loss
arose from a covered peril, but under an "all risks" policy the
burden is not on the insured to prove the precise cause of loss or
damage for which it seeks compensation. The insured under an "all
risks insurance policy" has the initial burden of proving that the
cargo was in good condition when the policy attached and that the
cargo was damaged when unloaded from the vessel; thereafter,
the burden then shifts to the insurer to show the exception to the
coverage. - none was shown = liable
 Section 13 of the Insurance Code defines insurable interest in
property as every interest in property, whether real or personal, or
any relation thereto, or liability in respect thereof, of such nature
that a contemplated peril might directly damnify the insured.
 As vendee/consignee of the goods in transit has such existing
interest. His interest over the goods is based on the perfected
contract of sale. The perfected contract of sale between him and
the shipper of the goods operates to vest in him an equitable title
even before delivery or before be performed the conditions of the
sale. The contract of shipment, whether under F.O.B., C.I.F., or C.
& F. as in this case, is immaterial in the determination of whether
the vendee has an insurable interest or not in the goods in transit.
 Article 1523 of the Civil Code provides that where, in pursuance of
a contract of sale, the seller is authorized or required to send the
goods to the buyer, delivery of the goods to a carrier, whether
named by the buyer or not, for, the purpose of transmission to the
buyer is deemed to be a delivery of the goods to the buyer, the
exceptions to said rule not obtaining in the present case. The Court
has heretofore ruled that the delivery of the goods on board the
carrying vessels partake of the nature of actual delivery since, from
that time, the foreign buyers assumed the risks of loss of the goods
and paid the insurance premium covering them
 C & F contracts are shipment contracts. The term means that the
price fixed includes in a lump sum the cost of the goods and freight
to the named destination. It simply means that the seller must pay
the costs and freight necessary to bring the goods to the named
destination but the risk of loss or damage to the goods is
transferred from the seller to the buyer when the goods pass the
ship's rail in the port of shipment.
 Moreover, the issue of lack of insurable interest was not among the
defenses averred in petitioners answer.

Cha v. Cha - Insurable Interest

Facts:

> Spouses Nilo Cha and Stella Uy-Cha, as lessees, entered into a
lease contract with CKS Development Corporation (CKS), as lessor.
> One of the stipulations of the one (1) year lease contract
states: "18. . . . The LESSEE shall not insure against fire the
chattels, merchandise, textiles, goods and effects placed at any stall or
store or space in the leased premises without first obtaining the
written consent and approval of the LESSOR. If the LESSEE obtain(s)
the insurance thereof without the consent of the LESSOR then the
policy is deemed assigned and transferred to the LESSOR for its own
benefit; . . ."

> Notwithstanding the above stipulation, the Cha spouses insured


against loss by fire their merchandise inside the leased premises for
Five Hundred Thousand (P500,000.00) with the United Insurance
without the written consent CKS.

> On the day that the lease contract was to expire, fire broke out
inside the leased premises. When CKS learned of the insurance earlier
procured by the Cha spouses (without its consent), it wrote the United
a demand letter asking that the proceeds of the insurance contract
(between the Cha spouses and United) be paid directly to CKS, based
on its lease contract with the Cha spouses.

> United refused to pay CKS, alleging that the latter had no insurable
interest. Hence, the latter filed a complaint against the Cha spouses
and United.

Issue:

Whether or not CKS can claim the proceeds of the fire insurance.

Held:NO. CKS has no insurable interest.

Sec. 18 of the Insurance Code provides:

"Sec. 18. No contract or policy of insurance on property shall be


enforceable except for the benefit of some person having an insurable
interest in the property insured."

A non-life insurance policy such as the fire insurance policy taken by


petitioner-spouses over their merchandise is primarily a contract of
indemnity. Insurable interest in the property insured must exist at the
time the insurance takes effect and at the time the loss occurs. The
basis of such requirement of insurable interest in property insured is
based on sound public policy: to prevent a person from taking out an
insurance policy on property upon which he has no insurable interest
and collecting the proceeds of said policy in case of loss of the
property.

In the present case, it cannot be denied that CKS has no insurable


interest in the goods and merchandise inside the leased premises
under the provisions of Section 17 of the Insurance Code which
provide:

"Section 17. The measure of an insurable interest in property is


the extent to which the insured might be damnified by loss of injury
thereof."

Therefore, CKS cannot, under the Insurance Code — a special law —


be validly a beneficiary of the fire insurance policy taken by the
petitioner-spouses over their merchandise. This insurable interest over
said merchandise remains with the insured, the Cha spouses. The
automatic assignment of the policy to CKS under the provision of the
lease contract previously quoted is void for being contrary to law
and/or public policy. The proceeds of the fire insurance policy thus
rightfully belong to the spouses Nilo Cha and Stella Uy-Cha (herein co-
petitioners). The insurer (United) cannot be compelled to pay the
proceeds of the fire insurance policy to a person (CKS) who has no
insurable interest in the property insured.

Sharuff and Co. v. Baloise Fire Insurance Co.- Proceeds of the Policy

Facts:

> Sharuff and Eskenazi were doing business under the firm name
Sharuff and Co.

> They insured their merchandise with Baloise. Later on, Sharuff and
Eskenazi entered into a contract of partnership and thereby changed
the firm name to Sharuff and Eskenazi.

> The merchandise insured was subsequently destroyed by


fire. Sharuff and Eskenazi filed their claim against the insurance
company.
> Baloise refused to pay on the ground that the policy was issued in
the name of Sharuff and Co. and not Sharuff and Eskenazi.

Issue:Whether or not the partnership can claim the proceeds of the


policy.

Held:Yes.

The subsequent partnership did not alter the composition of the


firm. The people involved are actually the same. Furthermore, such
change of firm name was not made to defraud the insurance company
or some other person.]

Sharuff & Co. V. Baloise Fire Insurance Co. (1937)


Lessons Applicable: Effect of Lack of Insurable Interest (Insurance)

Laws Applicable:

FACTS:
 Salomon Sharruf and Elias Eskenazi were doing business under the
firm name of Sharruf & Co. They insured their stocks with aloise
Fire Insurance Co., Sun Insurance Office Ltd., and Springfield
Insurance Co. raising it to P40,000. Elias Eskenazi having paid the
corresponding premiums
 Soon they changed the name of their partnership to Sharruf &
Eskenazi
 September 22, 1933: A fire ensued at their building at Muelle de la
Industria street where petroleum was spilt lasting 27 minutes
 Sharruf & Co. claimed 40 cases when only 10 or 11 partly burned
and scorched cases were found
 RTC: ordered Baloise Fire Insurance Co., Sun Insurance Office Ltd.,
and Springfield Insurance Co., to pay the partners Salomon Sharruf
and Elias Eskenazi P40,000 plus 8% interest
ISSUE: W/N Sharruf & Eskenazi has juridical personality and insurable
interest
HELD: YES. Reversd. Insurance companies are absolved.
 It does not appear that in changing the title of the partnership they
had the intention of defrauding the insurance companies
 fire which broke out in the building at Nos. 299-301 Muelle de la
Industria, occupied by Sharruf & Eskenazi but no evidence
sufficient to warrant a finding that they are responsible for the fire
 So great is the difference between the amount of articles insured,
which the plaintiffs claim to have been in the building before the
fire, and the amount thereof shown by the vestige of the fire to
have been therein, that the most liberal human judgment can not
attribute such difference to a mere innocent error in estimate or
counting but to a deliberate intent to demand of the insurance
companies payment of an indemnity for goods not existing at the
time of the fire, thereby constituting the so-called "fraudulent
claim" which, by express agreement between the insurers and the
insured, is a ground for exemption of the insurers from civil liability
 acted in bad faith in presenting a fraudulent claim, they are not
entitled to the indemnity claimed
 when the partners of a general partnership doing business under
the firm name of "Sharruf & Co." obtain insurance policies issued to
said firm and the latter is afterwards changed to "Sharruf &
Eskenazi", which are the names of the same and only partners of
said firm "Sharruf & Co.", continuing the same business, the new
firm acquires the rights of the former under the same policies;

Garcia v. HongKong Fire and Marine Insurance Co. - Wrong Policy

Facts:

> Garcia had his merchandise insured by Hongkong Fire and Marine
Insurance Co.

> The insurance company however made a mistake and issued a


policy covering the building where the merchandise was stored. (The
building was not owned by Garcia)
> The policy was written in English, of which Garcia was ignorant, so
he could not have noticed the error of the insurance company.

> Said policy was later on assigned by Garcia to PNB to secure a


loan. PNB acknowledged receipt of said policy, referring to it as a
policy covering the merchandise.

> The insurance company made the necessary endorsements to PNB.

> The building which housed the merchandise was later razed by
fire. The insurance company refused to pay due to the fact that the
policy indicates insurance on the building and not on the merchandise.

Issue:Whether or not Garcia can collect.

Held:YES.

The defense of the insurer is purely technical. The mistake was


obviously on the part of the insurer when it issued a wrong policy. It
cannot deny such allegation due to the fact that it even confirmed with
PNB the nature of said policy when it was endorsed. Garcia could not
have noticed the mistake due to his ignorance of the English language.

Garcia V. Hongkong Fire & Marine Insurance Co. (1923)

Lessons Applicable: Effect of Lack of Insurable Interest (Insurance)


Laws Applicable:
FACTS:

 August 30, 1919: Garcia executed a mortgage to the Philippine


National Bank on the merchandise allegedly insured by Hongkong
Fire & Marine Insurance Co. and with the consent of the latter
endorsed the policy to PNB
 PNB informed Hongkong Fire through exchange of
letters. Hongkong failed to notify PNB or Garcia that it was for the
building and not the merchandise.
 February 6, 1920: Fire took place and destroyed the merchandise
so Garcia filed a claim which was refused.
 RTC: favored Garcia
ISSUE: W/N
HELD: the lower court is affirmed

 as a matter of fair dealing, it should have notified the Bank that the
policy was on the building. It will be noted that the letters in
question were all written several months before the fire.
 Under these circumstances it seems clear and manifest that the
insured, as well as the manager of the National Bank at Legaspi,
who was interested in the policy, because the same secured a loan
of P6,000 made to Domingo Garcia, and the corporation of Wise &
Co., Ltd., which represented the insurance company, have been in
the belief that it was not the building but the merchandise that was
insured, for the reason that none of them paid attention to the
context of the policy.
 Tai Tong Chuache & Co. V. Insurance Commission (1988)
Lessons Applicable: When Insurable Interest Must Exist (Insurance)
Laws Applicable:
FACTS:

 Azucena Palomo bought a parcel of land and building from Rolando


Gonzales and assumed a mortgage of the building in favor of S.S.S.
which was insured with S.S.S. Accredited Group of Insurers
 April 19, 1975: Azucena Palomo obtained a loan from Tai Tong
Chuache Inc. in the amount of P100,000 and to secure it, the land
and building was mortgaged
 June 11, 1975: Pedro Palomo secured a Fire Insurance
Policy covering the building for P50,000 with Zenith Insurance
Corporation
 July 16, 1975: another Fire Insurance policy was procured
from Philippine British Assurance Company, covering the same
building for P50,000 and the contents thereof for P70,000
 Before the occurrence of the peril insured against the Palomos had
already paid their credit due the
 July 31, 1975: building and the contents were totally razed by fire
 Palomo was able to claim P41,546.79 from Philippine British
Assurance Co., P11,877.14 from Zenith Insurance Corporation and
P5,936.57 from S.S.S. Group of Accredited Insurers but Travellers
Multi-Indemnity refused so it demanded the balance from the other
three but they refused so they filed against them
 Insurance Commission, CFI: absolved Travellers on the basis that
Arsenio Cua was claiming and NOT Tai Tong Chuache
 Palomo Appealed
 Travellers reasoned that the policy is endorsed to Arsenio
Chua, mortgage creditor
 Tai Tong Chuache & Co. filed a complaint in intervention claiming
the proceeds of the fire Insurance Policy issued by travellers
 affirmative defense of lack of insurable interest that before the
occurrence of the peril insured against the Palomos had already
paid their credit due the petitioner
ISSUE: W/N Tai Tong Chuache & Co. has insurable interest
HELD: YES. Travellers Multi-Indemnity Corporation to pay Tai Tong
Chuache & Co.
 when the creditor is in possession of the document of credit, he
need not prove non-payment for it is presumed
 The validity of the insurance policy taken b petitioner was not
assailed by private respondent. Moreover, petitioner's claim that
the loan extended to the Palomos has not yet been paid was
corroborated by Azucena Palomo who testified that they are still
indebted to herein petitioner
 Chua being a partner of petitioner Tai Tong Chuache & Company is
an agent of the partnership. Being an agent, it is understood that
he acted for and in behalf of the firm
 Upon its failure to prove the allegation of lack of insurable interest
on the part of the petitioner, Travellers must be held liable
CONCEALMENT/MISREPRESENTATION

Great Pacific Life Assurance Corp. V. CA (1999)

Lessons Applicable:

 Credit in Life and Health Insurance (Insurance)


 Mortgagor (Insurance)
Laws Applicable: Sec. 8 of Insurance Code
FACTS:
 A contract of group life insurance was executed between Great
Pacific Life Assurance Corporation Grepalife) and Development
Bank of the Philippines (DBP)
 Grepalife agreed to insure the lives of eligible housing loan
mortgagors of DBP
 November 11, 1983: Dr. Wilfredo Leuterio, a physician and a
housing debtor of DBP applied for membership in the group life
insurance plan
 Dr. Leuterio answered questions concerning his health condition as
follows:

“7. Have you ever had, or consulted, a physician for a heart


condition, high blood pressure, cancer, diabetes, lung, kidney or
stomach disorder or any other physical impairment?
Answer: No. If so give details ___________.
8. Are you now, to the best of your knowledge, in good health?
Answer: [ x ] Yes [ ] No.”[4]

 November 15, 1983: Grepalife issued Certificate No. B-18558, as


insurance coverage of Dr. Leuterio, to the extent of his DBP
mortgage indebtedness amounting to P86,200
 August 6, 1984: Dr. Leuterio died due to “massive cerebral
hemorrhage.”
 DBP submitted a death claim to Grepalife
 Grepalife denied the claim alleging that Dr. Leuterio was not
physically healthy when he applied
 RTC: Favored Medarda V. Leuterio (widow) and held Grepalife
(insurer) liable to pay DBP (creditor of the insured Dr. Wilfredo
Leuterio)
 CA sustained
ISSUE:
1. W/N DBP has insurable interest as creditor - YES
2. W/N Grepalife should be held liable - YES

HELD:
1. YES
 In this type of policy insurance, the mortgagee is simply an
appointee of the insurance fund, such loss-payable clause does not
make the mortgagee a party to the contract
 Section 8 of the Insurance Code provides:

“Unless the policy provides, where a mortgagor of property effects


insurance in his own name providing that the loss shall be payable to
the mortgagee, or assigns a policy of insurance to a mortgagee, the
insurance is deemed to be upon the interest of the mortgagor, who
does not cease to be a party to the original contract, and any act of
his, prior to the loss, which would otherwise avoid the insurance, will
have the same effect, although the property is in the hands of the
mortgagee, but any act which, under the contract of insurance, is to
be performed by the mortgagor, may be performed by the mortgagee
therein named, with the same effect as if it had been performed by the
mortgagor.”
 The insured Dr. Wilfredo Leuterio did not cede to the mortgagee all
his rights or interests in the insurance. When Grepalife denied
payment, DBP collected the debt from the mortgagor and took the
necessary action of foreclosure on the residential lot of Dr. Wilfredo
Leuterio
 Insured may be regarded as the real party in interest, although he
has assigned the policy for the purpose of collection, or has
assigned as collateral security any judgment he may obtain
2. YES
 medical findings were not conclusive because Dr. Mejia did not
conduct an autopsy
 widow who was not even sure if the medicines taken by Dr.
Leuterio were for hypertension
 Grepalife failed to establish that there was concealment made by
the insured, hence, it cannot refuse payment of the claim
 fraudulent intent on the part of the insured must be established to
entitle the insurer to rescind the contract. Misrepresentation as a
defense of the insurer to avoid liability is an affirmative defense
and the duty to establish such defense by satisfactory and
convincing evidence rests upon the insurer
 The policy states that upon receipt of due proof of the Debtor’s
death during the terms of this insurance, a death benefit in the
amount of P86,200.00 shall be paid. In the event of the debtor’s
death before his indebtedness with the creditor shall have been
fully paid, an amount to pay the outstanding indebtedness shall
first be paid to the Creditor and the balance of the Sum Assured, if
there is any shall then be paid to the beneficiary/ies designated by
the debtor.
 DBP foreclosed in 1995 their residential lot, in satisfaction of
mortgagor’s outstanding loan
 insurance proceeds shall inure to the benefit of the heirs of the
deceased person or his beneficiaries
 Equity dictates that DBP should not unjustly enrich itself at the
expense of another (Nemo cum alterius detrimenio
protest). Hence, it cannot collect the insurance proceeds, after it
already foreclosed on the mortgage

Saturnino v. Philamlife - False Representation

Facts:

> 2 months prior to the insurance of the policy, Saturnino was


operated on for cancer, involving complete removal of the right breast,
including the pectoral muscles and the glands, found in the right
armpit.

> Notwithstanding the fact of her operation, Saturnino did not make a
disclosure thereof in her application for insurance.

> She stated therein that she did not have, nor had she ever had,
among others listed in the application, cancer or other tumors; that
she had not consulted any physician, undergone any operation or
suffered any injury within the preceding 5 years.

> She also stated that she had never been treated for, nor did she
ever have any illness or disease peculiar to her sex, particularly of the
breast, ovaries, uterus and menstrual disorders.

> The application also recited that the declarations of Saturnino


constituted a further basis for the issuance of the policy.

Issue:

Whether or not the insured made such false representation of material


facts as to avoid the policy.

Held:YES.

There can be no dispute that the information given by her in the


application for insurance was false, namely, that she never had cancer
or tumors or consulted any physician or undergone any operation
within the preceding period of 5 years.
The question to determine is: Are the facts then falsely represented
material? The Insurance Law provides that “materiality is to be
determined not by the event, but solely by the probable and
reasonable influence of the facts upon the party to whom the
communication is due, in forming his estimate of the proposed
contract, or making his inquiries.

The contention of appellants is that the facts subject of the


representation were not material in view of the non-medical nature of
the insurance applied for, which does away with the usual requirement
of medical examination before the policy is issued. The contention is
without merit. If anything, the waiver of medical examination renders
even more material the information required of the applicant
concerning previous condition of health and diseases suffered, for such
information necessarily constitutes an important factor which the
insurer takes into consideration in deciding whether to issue the policy
or not.

Appellants also contend that there was no fraudulent concealment of


the truth inasmuch as the insured herself did not know, since her
doctor never told her, that the disease for which she had been
operated on was cancer. In the first place, concealment of the fact of
the operation itself was fraudulent, as there could not have been any
mistake about it, no matter what the ailment.

Secondly, in order to avoid a policy, it is not necessary to show actual


fraud on the part of the insured. In this jurisdiction, concealment,
whether intentional or unintentional entitled the insurer to rescind the
contract of insurance, concealment being defined as “negligence to
communicate that which a party knows and ought to communicate.”
The basis of the rule vitiating the contract in cases of concealment is
that it misleads or deceives the insurer into accepting the risk, or
accepting it at a rate of premium agreed upon. The insurer, relying
upon the belief that the insured will disclose every material fact within
his actual or presumed knowledge, is misled into a belief that the
circumstances withheld does not exist, and he is thereby induced to
estimate the risk upon a false basis that it does not exist.
Musngi v. West Coast Life Assurance Co.- False Representation

Facts:

> Arsenio Garcia was insured by West Coast twice in 1931. In both
policies, he was asked to answer the question: “what physician or
practitioners have you consulted or been treated by, and for what
illness or ailment?

> In both policies, he answered in the negative. It turned out that


from 1929 to 1939, he went to see several physicians for a number of
ailments. So when he died in 1942, the company refused to pay the
proceeds of the insurance.

Issue:

Whether or not the answer given by Arsenio in the policies justifies the
company’s refusal to pay?

Held:YES.

Aresenio knew that he was suffering from a number of ailments, yet,


he concealed this. Such concealment and his false statements
constituted fraud, because the insurance company by reasons of such
statement accepted the risk which it would otherwise have rejected.

Sun Life v. CA - Concealment in Insurance

Facts:

> On April 15, 1986, Bacani procured a life insurance contract for
himself from Sun Life. He was issued a life insurance policy with
double indemnity in case of accidental death. The designated
beneficiary was his mother, Bernarda.

> On June 26, 1987, the insured died in a plane crash. Bernarda
Bacani filed a claim with Sun Life, seeking the benefits of the
insurance. Sun Life conducted an investigation and its findings
prompted it to reject the claim.
> Sun Life discovered that 2 weeks prior to his application, Bacani
was examined and confined at the Lung Center of the Philippines,
where he was diagnosed for renal failure. During his confinement, the
deceased was subjected to urinalysis, ultra-sonography and
hematology tests. He did not reveal such fact in his application.

> In its letter, Sun Life informed Berarda, that the insured did not
disclosed material facts relevant to the issuance of the policy, thus
rendering the contract of insurance voidable. A check representing the
total premiums paid in the amount of P10,172.00 was attached to said
letter.

> Bernarda and her husband, filed an action for specific performance
against Sun Life. RTC ruled for Bernarda holding that the facts
concealed by the insured were made in good faith and under the belief
that they need not be disclosed. Moreover, it held that the health
history of the insured was immaterial since the insurance policy was
"non-medical." CA affirmed.

Issue:

Whether or not the beneficiary can claim despite the concealment.

Held:NO

Section 26 of the Insurance Code is explicit in requiring a party to a


contract of insurance to communicate to the other, in good faith, all
facts within his knowledge which are material to the contract and as to
which he makes no warranty, and which the other has no means of
ascertaining.

Materiality is to be determined not by the event, but solely by the


probable and reasonable influence of the facts upon the party to whom
communication is due, in forming his estimate of the disadvantages of
the proposed contract or in making his inquiries (The Insurance Code,
Sec 31)

The terms of the contract are clear. The insured is specifically required
to disclose to the insurer matters relating to his health. The
information which the insured failed to disclose were material and
relevant to the approval and the issuance of the insurance policy. The
matters concealed would have definitely affected petitioner's action on
his application, either by approving it with the corresponding
adjustment for a higher premium or rejecting the same. Moreover, a
disclosure may have warranted a medical examination of the insured
by petitioner in order for it to reasonably assess the risk involved in
accepting the application.

Thus, "good faith" is no defense in concealment. The insured's failure


to disclose the fact that he was hospitalized for two weeks prior to
filing his application for insurance, raises grave doubts about his
bonafides. It appears that such concealment was deliberate on his
part.

Sunlife v CA G.R. No. 105135 June 22, 1995


Facts:
Robert John B. Bacani procured a life insurance contract for himself from
Sunlife. He was issued a policy for P100,000.00, with double indemnity
in case of accidental death. The designated beneficiary was his mother,
Bernarda Bacani.
The insured died in a plane crash. Respondent Bernarda Bacani filed a
claim with petitioner, seeking the benefits of the insurance policy taken
by her son. Petitioner conducted an investigation and its findings
prompted it to reject the claim.
Sunlife informed Bacani that the insured did not disclose material facts
relevant to the issuance of the policy, thus rendering the contract of
insurance voidable. A check representing the total premiums paid in the
amount of P10,172.00 was attached to said letter.
Petitioner claimed that the insured gave false statements in his
application. The deceased answered claimed that he consulted a Dr.
Raymundo of the Chinese General Hospital for cough and flu
complications. The other questions were answered in the negative.
Petitioner discovered that two weeks prior to his application for
insurance, the insured was examined and confined at the Lung Center
of the Philippines, where he was diagnosed for renal failure. During his
confinement, the deceased was subjected to urinalysis tests.
Bernarda Bacani and her husband filed an action for specific
performance against petitioner with the RTC. The court ruled in favor of
the spouses and ordered Sunlife to pay P100,000.00.
In ruling for private respondents, the trial court concluded that the facts
concealed by the insured were made in good faith and under a belief
that they need not be disclosed. The court also held that the medial
history was irrelevant because it wasn’t medical insurance.
The Court of Appeals affirmed the decision of the trial court. The
appellate court ruled that petitioner cannot avoid its obligation by
claiming concealment because the cause of death was unrelated to the
facts concealed by the insured. Petitioner's motion for reconsideration
was denied. Hence, this petition.

Issue: WON the insured was guilty of misrepresentation which made the
contract void.

Held: Yes. Petition dismissed.

Ratio:
Section 26 of The Insurance Code required a party to a contract of
insurance to communicate to the other, in good faith, all facts within his
knowledge which are material to the contract and as to which he makes
no warranty, and which the other has no means of ascertaining.
“A neglect to communicate that which a party knows and ought to
communicate, is called concealment.”
“Materiality is to be determined not by the event, but solely by the
probable and reasonable influence of the facts upon the party to whom
communication is due, in forming his estimate of the disadvantages of
the proposed contract or in making his inquiries.”
The terms of the contract are clear. The insured is specifically required
to disclose to the insurer matters relating to his health.
The information which the insured failed to disclose were material and
relevant to the approval and issuance of the insurance policy. The
matters concealed would have definitely affected petitioner's action on
his application, either by approving it with the corresponding adjustment
for a higher premium or rejecting the same. Moreover, a disclosure may
have warranted a medical examination of the insured by petitioner in
order for it to reasonably assess the risk involved in accepting the
application.
Vda. de Canilang v. Court of Appeals- materiality of the information
withheld does not depend on the state of mind of the insured. Neither
does it depend on the actual or physical events which ensue.
“Good faith" is no defense in concealment. The insured's failure to
disclose the fact that he was hospitalized raises grave doubts about his
eligibility. Such concealment was deliberate on his part.
The argument, that petitioner's waiver of the medical examination of
the insured debunks the materiality of the facts concealed, is untenable.
Saturnino v. Philippine American Life Insurance " . . . the waiver of a
medical examination [in a non-medical insurance contract] renders even
more material the information required of the applicant concerning
previous condition of health and diseases suffered, for such information
necessarily constitutes an important factor which the insurer takes into
consideration in deciding whether to issue the policy or not . . . "
Anent the finding that the facts concealed had no bearing to the cause
of death of the insured, it is well settled that the insured need not die of
the disease he had failed to disclose to the insurer. It is sufficient that
his non-disclosure misled the insurer in forming his estimates of the
risks of the proposed insurance policy or in making inquiries as held in
Henson.

Ng Gan Zee v. Asian Crusader Life - Imperfection in the Application


Form

Facts:

> In 1962, Kwon Nam applied for a 20yr endowment insurance on his
life with his wife, Ng Gan Zee as the beneficiary.

> He stated in his application that he was operated on for tumor of


the stomach associated with ulcer.

> In 1963, Kwong died of cancer of the liver with metastasis. Asian
refused to pay on the ground of alse information.

> It was found that prior to his application, Kwong was diagnosed to
have peptic ulcers, and that during the operation what was removed
from Kwong’s body was actually a portion of the stomach and not
tumor.

Issue:
Whether or not the contract may be rescinded on the ground of the
imperfection in the application form.

Held:NO.

Kwong did not have sufficient knowledge as to distinguish between a


tumor and a peptic ulcer. His statement therefore was made in good
faith. Asian should have made an inquiry as to the illness and
operation of Kwong when it appeared on the face of the application
that a question appeared to be imperfectly answered. Asian’s failure
to inquire constituted a waiver of the imperfection in the answer.

Ng v Asian Crusader G.R. No. L-30685 May 30, 1983

Facts:
Kwong Nam applied for a 20-year endowment insurance on his life for
the sum of P20,000.00, with his wife, appellee Ng Gan Zee as
beneficiary. On the same date, Asian Crusader, upon receipt of the
required premium from the insured, approved the application and issued
the corresponding policy. Kwong Nam died of cancer of the liver with
metastasis. All premiums had been paid at the time of his death.
Ng Gan Zee presented a claim for payment of the face value of the
policy. On the same date, she submitted the required proof of death of
the insured. Appellant denied the claim on the ground that the answers
given by the insured to the questions in his application for life insurance
were untrue.
Appellee brought the matter to the attention of the Insurance
Commissioner. The latter, after conducting an investigation, wrote the
appellant that he had found no material concealment on the part of the
insured and that, therefore, appellee should be paid the full face value
of the policy. The company refused to settle its obligation.
Appellant alleged that the insured was guilty of misrepresentation when
he answered "No" to the following question appearing in the application
for life insurance-
Has any life insurance company ever refused your application for
insurance or for reinstatement of a lapsed policy or offered you a policy
different from that applied for? If, so, name company and date.
The lower court ruled against the company on lack of evidence.
Appellant further maintains that when the insured was examined in
connection with his application for life insurance, he gave the appellant's
medical examiner false and misleading information as to his ailment and
previous operation. The company contended that he was operated on
for peptic ulcer 2 years before the policy was applied for and that he
never disclosed such an operation.

Issue: WON Asian Crusader was deceived into entering the contract or
in accepting the risk at the rate of premium agreed upon because of
insured's representation?

Held: No. Petition dismissed.

Ratio:
Section 27 of the Insurance Law:
Sec. 27. Such party a contract of insurance must communicate to the
other, in good faith, all facts within his knowledge which are material to
the contract, and which the other has not the means of ascertaining,
and as to which he makes no warranty.
"Concealment exists where the assured had knowledge of a fact
material to the risk, and honesty, good faith, and fair dealing requires
that he should communicate it to the assurer, but he designedly and
intentionally withholds the same."
It has also been held "that the concealment must, in the absence of
inquiries, be not only material, but fraudulent, or the fact must have
been intentionally withheld."
Fraudulent intent on the part of the insured must be established to
entitle the insurer to rescind the contract. And as correctly observed by
the lower court, "misrepresentation as a defense of the insurer to avoid
liability is an 'affirmative' defense. The duty to establish such a defense
by satisfactory and convincing evidence rests upon the defendant. The
evidence before the Court does not clearly and satisfactorily establish
that defense."
It bears emphasis that Kwong Nam had informed the appellant's medical
examiner of the tumor. His statement that said tumor was "associated
with ulcer of the stomach" should be construed as an expression made
in good faith of his belief as to the nature of his ailment and operation.
While the information communicated was imperfect, the same was
sufficient to have induced appellant to make further inquiries about the
ailment and operation of the insured.
Section 32 of Insurance Law:
Section 32. The right to information of material facts maybe waived
either by the terms of insurance or by neglect to make inquiries as to
such facts where they are distinctly implied in other facts of which
information is communicated.
Where a question appears to be not answered at all or to be imperfectly
answered, and the insurers issue a policy without any further inquiry,
they waive the imperfection of the answer and render the omission to
answer more fully immaterial.
The company or its medical examiner did not make any further inquiries
on such matters from the hospital before acting on the application for
insurance. The fact of the matter is that the defendant was too eager to
accept the application and receive the insured's premium. It would be
inequitable now to allow the defendant to avoid liability under the
circumstances."

FRANCISCA EGUARAS v THE GREAT EASTERN LIFE ASSURANCE


COMPANY, LTD

Doctrine

A contract is deceitful, for the execution whereof the consent of one of


the parties has been secured by means of fraud, because he was
persuaded by words or insidious machinations, statements of false
promises, and a defective consent wrung from him, even though such
do not constitute estafa or any other criminal act subject to the penal
law.

Summary

Dominador took out a 5000 peso insurance policy from defendant


through its agent, Ponciano Remigio. He died a month later and it
turns out that he was not in good health when he applied for the
policy. Remigio had full knowledge of his condition so in order to close
the deal, he presented another person pretending to be Dominador
during the required medical examination. Because of this, the
insurance company refused to yield to the claims of the beneficiary of
the policy, Francisca Eguaras, Dominador’s mother in law. The SC
ruled that because the contract was fraudulently closed, it is thus void
and so Francisca cannot claim the proceeds thereof. Facts Dominador
Albay, petitioner’s son-in-law applied to the defendant insurance
company for his life to be insured for P5,000 and naming petitioner as
the beneficiary. The company accepted the application but while the
policy was in force, Dominador died. When Francisca tried to claim
benefits as beneficiary, the company refused to release the funds so
Francisca filed a complaint in court. The TC ruled in favor of Francisca
and sentenced the company to pay P5,000. Upon appeal, the
company argued that it refused to pay the claimant because they
found out that the policy had been fraudulently applied for. It found
that their agent, Ponciano Remigio who sold the policy to Dominador
knew that the latter had a serious medical condition before application.
In the application, Dominador had answered that he had a “good
constitution” health wise. Remigio then engaged the services of one
Dr. Jose Vidal supposedly to examine Dominador but because of his
knowledge of the latter’s less than normal medical standing, he
presented a different person for physical examination. This person,
Castor Garcia was examined in his place and was found to be in good
health. On this basis, the doctor recommended that the “Dominador
Albay” that he had examined was an insurable risk and it was because
of this that the company had issued the policy. It must be noted that
Ponciano had full knowledge of the fact that Dominador was not in
good health and had later died of intestinal occlusion after 3 days of
illness and medical attendance. Despite a previous offer for P600 from
Remigio to testify otherwise, Dr. Vidal confirmed the identity of the
person purporting to be Dominador during trial. Several witnesses also
testified regarding the seriousness of Dominador’s health prior to his
death such as Dr. Gertrudo Reyes and one Jose Valencia.

Issues

W/N the insurance company should pay Francisca the benefits she was
entitled to

Ratio

(NO) To further prove that fraud had been resorted to, the SC
compared the signatures of the real Dominador from previous
documents and that of Castor Garcia on the medical form. Dominador
had a firm and solid handwriting, culled from years of education in
American schools. They further examined Dominador’s signature on
his electoral oath, which was signed before the board of election
inspectors and duly identified by the chairman of said board. It found
that it matched the signature on the provisional receipt issued by the
company upon his application for the policy. In comparison, the
signature on the medical forms and other subsequent documents were
found to be radically different so it can be concluded that it was done
by a different person. Also, his age on the application and insurance
policy was 40 years old in 1912 while in his personal cedulas later on,
he was only 32 in 1911. Article 1269, CC: There is deceit when by
words or insidious machinations on the part of one of the contracting
parties the other is induced to execute a contract which without them
he would not have made. It is essential that deceit be made prior to
or contemporaneous with the consent necessary to perfect a contract.
The company in this case was induced by the result of the medical
examination and the recommendation of Remigio. The contract
therefore, is void and ineffective in accordance with the provisions of
the civil code. Had Dominador not been substituted, the physician
would not have been able to identify Castor Garcia as the person he
examined on trial. Remigio had resorted to fraudulent means in order
to secure the payment for the policy and although it does not
constitute a crime, the contract is still void. As a result, the company is
not obliged to pay Francisca the proceeds from the policy. NOTE:
Although Remigio and Eguaras were previously tried and acquitted for
the crime of estafa, the acquittal does not produce the effect of res
judicata. The contract was still void for the vitiation of the company’s
consent through fraud. (See Doctrine) Held Judgment appealed from
reversed.

Argente v. West Coast Life Insurance Co.- Misrepresentation

Facts:

> A joint life insurance policy was issued to Bernardo Argente and his
wife Vicenta upon payment of premium, by West Coast.

> On Nov. 18, 1925, during the effectivity of the policy, Vicenta died
of cerebral apoplexy. Thereafter, Bernardo claimed payment but was
refused.
> It is admitted that in the Medical Examiner’s report, Vicenta, in
response to the question asked by the medical examiner, her replies
were as follows:

o “How frequently do you use beer, wine, spirits and other


intoxicants?” she answered “beer only in small quantities”.

o “What physician have you consulted or been treated by within the


last 5 years and for what illness or ailment?” she answered “none”

> It is however, not disputed that in 1924, Vicenta was taken to a


hospital for what was first diagnosed as alcoholism and later changed
to manic-depressive psychosis and then again changed to
pscyhonuerosis.

Issue:

Whether or not on the basis of the misrepresentations of Vicenta,


Bernardo is barred from recovery.

Held:YES.

The court found that the representations made by Vicenta in his


application for life insurance were false with respect to her state of
health and that she knew and was aware that the representations so
made by her were false. In an action on a life insurance policy where
the evidence conclusively shows that the answers to questions
concerning diseases were untrue, the truth or falsity of the answer
becomes the determining factor.

If the policy was procured by fraudulent misrepresentations, the


contract of insurance apparently set forth therein was never legally
existent. It can be fairly assumed that had the true facts been
disclosed by the insured, the insurance would never have been
granted.

Insular Life v. Feliciano - Concealment

Facts:

> Evaristo Feliciano filed an application with Insular Life upon the
solicitation of one of its agents.
> It appears that during that time, Evaristo was already suffering
from tuberculosis. Such fact appeared during the medical exam, but
the examiner and the company’s agent ignored it.

> After that, Evaristo was made to sign an application form and
thereafter the blank spaces were filled by the medical examiner and
the agent making it appear that Evaristo was a fit subject of insurance.
(Evaristo could not read and understand English)

> When Evaristo died, Insular life refused to pay the proceeds
because of concealment.

Issue:Whether or not Insular Life was bound by their agent’s acts.

Held:Yes.

The insurance business has grown so vast and lucrative within the past
century. Nowadays, even people of modest means enter into
insurance contracts. Agents who solicit contracts are paid large
commissions on the policies secured by them. They act as general
representatives of insurance companies.

IN the case at bar, the true state of health of the insured was
concealed by the agents of the insurer. The insurer’s medical
examiner approved the application knowing fully well that the
applicant was sick. The situation is one in which of two innocent
parties must bear a loss for his reliance upon a third person. In this
case, it is the one who drafted and accepted the policy and
consummated the contract. It seems reasonable that as between the
two of them, the one who employed and gave character to the third
person as its agent should be the one to bear the loss. Hence, Insular
is liable to the beneficiaries.

Das könnte Ihnen auch gefallen